Civil Procedure Flashcards

You may prefer our related Brainscape-certified flashcards:
1
Q

Federal question jurisdiction requires that the federal question appear on a fair reading of the complaint.

Because the plaintiff did not invoke federal law (his claims arose under state law), there will be no federal question jurisdiction.

Example?

A

D, school argued that it fired an employee based on a federal regulation prohibiting schools from employing anyone who had been convicted.

BUT

P filed suit for unlawful discrimination based on state law.

How well did you know this?
1
Not at all
2
3
4
5
Perfectly
2
Q

The citizenship for diversity purposes is determined …

A

…at the time the complaint was filed.

How well did you know this?
1
Not at all
2
3
4
5
Perfectly
3
Q

Federal courts have original jurisdiction over admiralty or maritime cases. This means that, …

A

…even where a plaintiff’s tort claims are grounded in state law, a federal court will have jurisdiction if the tort occurred on navigable waters or was caused by a vessel on navigable waters and the case has a “maritime nexus.” Having a maritime nexus means the incident at issue had a potentially disruptive impact on maritime commerce and a substantial relationship to traditional maritime activity.

How well did you know this?
1
Not at all
2
3
4
5
Perfectly
4
Q

Can attorney fees be included in the amount in controversy?

A

Yes.

How well did you know this?
1
Not at all
2
3
4
5
Perfectly
5
Q

Ordinarily, if a defendant is physically present in a jurisdiction and is served with process while there, the court’s exercise of personal jurisdiction over that defendant constitutional. It does not matter if the defendant’s presence at the time is temporary or unrelated to the lawsuit. However, …

A

the defendant’s presence must be voluntary and the plaintiff cannot have coaxed the defendant into the state under false pretenses.

i.e. Had the customer served the woman while she happened to be in the state on one of her own business trips, this would have been enough to provide the court with personal jurisdiction over her. Because the customer instead deceived the woman into entering the state in order to physically serve her, such service is improper and the court will not have personal jurisdiction over the woman.

How well did you know this?
1
Not at all
2
3
4
5
Perfectly
6
Q

Quasi in rem jurisdiction applies …

A

…where a court cannot exercise in personam jurisdiction due to there not being a sufficient long-arm statute in place, but where there is property in the forum state that is attached as part of the relief requested. There must still be minimum contacts and a finding that the exercise of jurisdiction does not offend traditional notions of fair play and substantial justice.

How well did you know this?
1
Not at all
2
3
4
5
Perfectly
7
Q

When analyzing whether a case should be dismissed under the doctrine of forum non conveniens, a court will …

A

…balance a number of private and public interests, which generally go to the inconvenience and expense that would be incurred by the parties, the court, and the community in conducting a trial in the forum.

Whether a jury might be biased is not one of the factors generally considered in a forum non conveniens analysis.

How well did you know this?
1
Not at all
2
3
4
5
Perfectly
8
Q

Federal circuit courts of appeal do not give deference to district courts’ interpretations of unsettled state law, but …

A

…review those decisions de novo.

How well did you know this?
1
Not at all
2
3
4
5
Perfectly
9
Q

Methods for a federal court to approach a question of unsettled state law.

A

Where a federal court is faced with a question of state law where the applicable law is uncertain or unclear, a federal court

(1) Under the doctrine of abstention, may abstain from hearing a question of unsettled state law.
(2) May certify its questions to the state court to obtain a ruling on the issue.
(3) May try to predict how the applicable state court would rule on the issue.

How well did you know this?
1
Not at all
2
3
4
5
Perfectly
10
Q

Courts may create federal common law in certain situations, pursuant to Clearfield Trust Co. v. U.S. [318 U.S. 360 (1943)]. One situation that a court may do so is …

A

..where Congress has passed a law which inadequately addresses the situation concerned.

How well did you know this?
1
Not at all
2
3
4
5
Perfectly
11
Q

If electronically stored information that should have been preserved in the anticipation or conduct of litigation is lost because a party failed to take reasonable steps to preserve it, and it cannot be restored or replaced through additional discovery, the court:

A

(1) upon finding prejudice to another party from loss of the information, may order measures no greater than necessary to cure the prejudice; or (2) only upon finding that the party acted with the intent to deprive another party of the information’s use in the litigation, may: (A) presume that the lost information was unfavorable to the party; (B) instruct the jury that it may or must presume the information was unfavorable to the party; or (C) dismiss the action or enter a default judgment.

How well did you know this?
1
Not at all
2
3
4
5
Perfectly
12
Q

If a party fails to make required disclosures under Fed. R. Civ. P. 26(a), the court may sanction that party by …

A

…requiring the party to pay the reasonable costs, including attorney fees, caused by the delay, informing the jury of the failure to produce the evidence earlier, and any other sanctions.

How well did you know this?
1
Not at all
2
3
4
5
Perfectly
13
Q

Typically, a plaintiff is required to make a demand for a jury trial within 14 days of being served with a notice of removal by the other party.

Exception?

A

Where, however, the state law did not require express demand, the parties do not have to demand a jury trial unless the court so orders it, or at a party’s request.

How well did you know this?
1
Not at all
2
3
4
5
Perfectly
14
Q

The court must give the parties a chance to object to the proposed jury instructions …

A

…before the final jury arguments and out of the jury’s hearing.

How well did you know this?
1
Not at all
2
3
4
5
Perfectly
15
Q

When must a party object to a proposed jury instruction?

A

At the time the court informs it of the proposed jury instruction, or at the earliest time to object.

How well did you know this?
1
Not at all
2
3
4
5
Perfectly
16
Q

Demand for a jury trial must be made …

A

…within 14 days after the service of the last pleading directed to the triable issue, or the right is waived.

How well did you know this?
1
Not at all
2
3
4
5
Perfectly
17
Q

On a motion for summary judgment, may a plaintiff may rely on evidence that would be inadmissible at trial to satisfy the production burden?

A

No.

How well did you know this?
1
Not at all
2
3
4
5
Perfectly
18
Q

Dismissal of a claim under Fed. R. Civ. P. 12(b)(6) is with or without prejudice?

A

with

v.

Where a plaintiff has voluntarily dismissed a case by filing a notice of dismissal before the defendant has filed an answer, then the dismissal will be without prejudice. The defendant here had served a motion to dismiss, which had not been ruled upon, and had not yet filed an answer when the plaintiff served her notice of dismissal, so the plaintiff may properly bring a new action.

How well did you know this?
1
Not at all
2
3
4
5
Perfectly
19
Q

Jurors - number, excusing, unanimous verdict

A

A jury must begin with between six and 12 members.

The parties may stipulate to having less than six jurors render a verdict in exceptional circumstances.

Jurors may be excused during the trial.

Pursuant to Fed. R. Civ. P. 48(b), a verdict must be unanimous.

Where a jury returns a verdict that is less than unanimous, the court may poll the jurors and then send them back to continue to deliberate.

How well did you know this?
1
Not at all
2
3
4
5
Perfectly
20
Q

Pursuant to Fed. R. Civ. P. 52(c), a court may enter a judgment against a party based on a partial finding on an issue, where the action may only be maintained on a favorable finding on that issue.

Example?

A

I.e.

The student’s First Amendment claim required that his religious beliefs be sincerely held, School made Rule 52 (c) motion, arguing student had failed to show his belief was sincere.

If the court grants the motion, however, such an order must be accompanied by findings of fact and conclusions of law.

How well did you know this?
1
Not at all
2
3
4
5
Perfectly
21
Q

The general rule is that interlocutory orders are not reviewable prior to final judgment, but this is not an absolute rule as the collateral order doctrine does permit interlocutory review in certain instances.

What is collateral order doctrine?

A

Interlocutory orders are generally unreviewable, unless the order:

1) conclusively determines the disputed question; 2) resolves an important issue completely separate from the merits of the action; and 3) is effectively unreviewable on appeal from a final judgment.

How well did you know this?
1
Not at all
2
3
4
5
Perfectly
22
Q

Where an interlocutory order is not otherwise reviewable under the collateral order doctrine, ….

A

… a judge may still certify the order for appeal if he or she indicates in the order that the order involved a controlling question of law as to which a substantial ground for difference of opinion exists and that an immediate appeal from the order may materially advance the termination of the litigation.

How well did you know this?
1
Not at all
2
3
4
5
Perfectly
23
Q

Appelate review - de novo, clear error

A

An appellate court will review conclusions of law de novo–meaning the court will conduct a non-deferential review–but will only disturb factual findings if they are clearly erroneous.

How well did you know this?
1
Not at all
2
3
4
5
Perfectly
24
Q

each party is limited to …. interrogatories

a party is entitled to …. depositions as a matter of right.

A

25

10

How well did you know this?
1
Not at all
2
3
4
5
Perfectly
25
Q

As a general rule, people do not have standing as taxpayers to challenge the manner in which the federal government spends tax dollars, because their interest is too remote.

Under an exception to this general rule, federal taxpayers have standing to challenge federal appropriation and spending measures if they can establish that the challenged measure:

A

(i) was enacted under the taxing and spending power of Congress; and

(ii) exceeds some specific limitation on the power. The only such limit found by the Court to date on the taxing power is
the Establishment Clause.

How well did you know this?
1
Not at all
2
3
4
5
Perfectly
26
Q

A federal court is required to analyze whether it has personal jurisdiction over a defendant in the same manner as a state court. Thus, a personal jurisdiction analysis is exactly
the same in federal court and in state court.

A

In other words, it is irrelevant whether the court is a
state court or a federal court when it comes to a personal jurisdiction question.

How well did you know this?
1
Not at all
2
3
4
5
Perfectly
27
Q

A small corporation manufactures and sells
widgets in State A and State B. It is a State A
corporation and it operates three stores from
which it sells its widgets—two are in the District
of State A and one is in the District of State B.
The corporation has no presence, sales, or operations
anywhere else. A consumer who resides in
the District of State C purchased a widget from
the corporation’s State B store. The consumer
was subsequently injured while using the widget
at his home. The consumer intends to file a
products liability action against the corporation
in federal district court.
In what district or districts is venue proper?

A

Federal venue in civil actions is proper in

(i) the district where any defendant resides, if all defendants
are residents of the state in which the district is located; and

(ii) the district in which a substantial part of the events or omissions giving rise to the claim occurred.

If there is no district anywhere in the United States that satisfies (i) or (ii), the action may be brought in a judicial district in which any defendant is subject to the court’s personal jurisdiction with respect to such action.

The District of State C (place of the injury) and the District of State B (place of purchase) are proper because substantial events giving rise to the claim occurred there.

A corporation is deemed to reside in any district in which it is subject to personal jurisdiction with respect to the action in question. The facts state that the manufacturer is a State A corporation and has its principal place of business in the District of State A. As a result, it is subject to personal jurisdiction there based on a general jurisdiction theory. Thus, the District of State A is a place of proper venue because that is where the manufacturer (the only defendant) resides for purposes of venue

How well did you know this?
1
Not at all
2
3
4
5
Perfectly
28
Q

A citizen of State A purchased life insurance
by mail from a State B insurance company.
The policy was the only one that the company
had ever sold in State A. The purchaser mailed
premiums from State A to State B for five years,
and then died. The insurance company refused
to pay the policy benefits. The purchaser’s
administrator sued the company in State A
state court. The state has a long arm statute that
grants a state court in personam jurisdiction
over a defendant who “contract[s] to insure any
person, property, or risk located within this State
at the time of the contracting.” The insurance
company argued that its only contact with State
A since it began its business was the purchaser’s
insurance policy, and that this single contact
does not meet the minimum required for the
exercise of in personam jurisdiction under International
Shoe.
How should the court rule on the minimum
contacts issue?

A

The court should rule for the purchaser’s administrator on the minimum contacts issue and exercise in personam jurisdiction over the insurance company.

Generally, fewer contacts will suffice for in personam jurisdiction if those contacts are directly related to the cause of action.
Here, even though the sale of the insurance policy constituted only a single contact, it is the contact at issue in the case. Thus, the exercise of jurisdiction would be constitutional.

How well did you know this?
1
Not at all
2
3
4
5
Perfectly
29
Q

A pedestrian filed an action against a driver
in federal district court, alleging negligence.
The attorney for the driver has interviewed an
eyewitness whose testimony will clearly indicate
that the driver was at fault.
Must the driver disclose the existence and
identity of the eyewitness to the pedestrian?

A

The driver must disclose the identity of the witness in response to an appropriate interrogatory.

Federal Rule 26(a) requires, as an initial disclosure, a party to reveal the name and contact information of individuals who are likely to have discoverable information and who the disclosing party may use to support his claims or defenses (unless the use would be solely for impeachment).

After initial disclosures are made, discovery proceeds, and the parties may continue with discovery of nonprivileged information that is relevant to any party’s claim or defense, including the names and contact information of any person who knows of any discoverable matter.

Here, the eyewitness would not need to be disclosed as an initial disclosure because the driver obviously will not use the eyewitness to support the driver’s claim or defense. However, the identity of the eyewitness would need to be disclosed eventually, assuming the pedestrian submits a proper
discovery request.

How well did you know this?
1
Not at all
2
3
4
5
Perfectly
30
Q

A deposition may be used at trial for any purpose if the deponent is at a distance greater than ….

A

…100 miles from the place of trial.

How well did you know this?
1
Not at all
2
3
4
5
Perfectly
31
Q

A man filed a negligence action against a
woman in federal district court, alleging that
the woman negligently caused a collision
between their vehicles and seeking compensatory
damages. The man served on the woman
a request that the woman produce a specified
electronic recording for inspection or copying.
The woman made the recording immediately
after the accident while waiting for the police to
arrive. She made the recording for the specific
purpose of preparing for litigation that she knew
would ensue. The recording contains the observations
of three witnesses to the accident. The
woman objected to the man’s request for production
of the recording, and the man filed a motion
to compel production of the recording.
Should the court order the woman to produce
the recording?

A

The court should not order the woman to produce the recording unless it finds that the man has substantial need for the recording and is unable to obtain the same information without undue hardship.

In general, a party’s work product made in anticipation of litigation is not discoverable.
However, the work product may be discovered if the opposing party can show substantial need
and that he cannot obtain the same information in an alternative way without undue hardship
.

How well did you know this?
1
Not at all
2
3
4
5
Perfectly
32
Q

Can you waive a demand for a jury trial?

A

Under Federal Rule 38, a party who desires a jury trial
(on some or all fact issues) must file a written demand with the court and serve it on the parties.
Failure to make such a demand within 14 days after the service of the last pleading directed to the jury-triable issue constitutes a waiver by that party of any right to trial by jury.

***

Although the woman could amend her complaint as of right within 21 days of service of the homeowner’s answer, she did not have the right to include in it a demand for jury trial

How well did you know this?
1
Not at all
2
3
4
5
Perfectly
33
Q

A driver collided with a bicyclist, severely
injuring her. The bicyclist sued the driver in
the federal court for State A, properly invoking
diversity of citizenship. In his answer, the driver
raised the defense of contributory negligence,
which is a complete defense under the law of
State A, where the accident occurred. The court
instructed the jury to return a general verdict
with answers to written questions. In its written
answers, the jury found that both the driver
and the bicyclist were negligent. Nonetheless, it
returned a general verdict awarding the bicyclist
damages for her medical expenses and for
her pain and suffering.Which of the following
options is NO T available to the court to redress
the inconsistency between the general verdict
and the jury’s finding of contributory negligence?

A

Under the Federal Rules, when the jury’s answers accompanying a general verdict are consistent with each other but are inconsistent with the general verdict, the court may

(i) enter a judgment that is consistent with the answers, disregarding the general verdict;
(ii) instruct the jury to deliberate further regarding its verdict and answers; or
(iii) order a new trial.

How well did you know this?
1
Not at all
2
3
4
5
Perfectly
34
Q

When a verdict shows on its face that the jury failed to follow the court’s instructions, …

A

…the verdict may be set aside, and either the jury will be asked to reconsider its verdict or a new trial will be ordered.

How well did you know this?
1
Not at all
2
3
4
5
Perfectly
35
Q

A manufacturer sold to a consumer an expensive
laser printer that never worked properly.
Therefore, the consumer never fully paid for
the printer. The manufacturer sued for specific
performance of the contract of sale of the printer.
The consumer filed a counterclaim for a breach
of warranty, asking for $85,000 in damages.
The consumer demanded a jury trial, but the
manufacturer objected.
Assuming that the demand for a jury trial
was timely made, how will the court rule on the
availability of a jury trial?

A

The court will grant a jury trial because the underlying dispute is legal in nature. The court will
look to the basic substance of the case to see if a jury trial is appropriate. Although the manufacturer’s
suit is equitable in nature, the consumer’s counterclaim for breach of warranty is an action
at law, in which a jury trial is available on demand.

How well did you know this?
1
Not at all
2
3
4
5
Perfectly
36
Q

The plaintiff sued the defendant in federal
court for breach of contract. The plaintiff
presented a detailed case in which she showed
the existence of the contract, its breach, and
damages. The defendant then presented his
defense, which consisted solely of evidence that
he regretted dealing with the plaintiff. At this
point in the trial, the plaintiff feels she should
prevail.
What should she move for at this point in the
litigation?

A

At this point in the litigation, when the opposing party has been fully heard, the appropriate motion is for judgment as a matter of law (formerly known in federal practice as a motion for
directed verdict). The evidence will be viewed in the light most favorable to the party opposing the motion, and will be granted if there is no legally sufficient evidentiary basis for a reasonable
jury to find in favor of that party.

Motion for judgment on the pleading and motion for summary judgment pretrial motions:

a motion for judgment on the pleadings considers no evidence outside the complaint, answer, etc.,

while a motion for summary judgment considers affidavits, discovery material, and other evidence submitted with
the motion.

How well did you know this?
1
Not at all
2
3
4
5
Perfectly
37
Q

the jury demand because a jury demand may be
withdrawn only if …

A

…all parties consent

***

There is no
timing specified in the federal rules for withdrawing a jury demand, but a jury demand can be
withdrawn only with the consent of all the parties. The 14-day period represents the general time
period for initially demanding a jury.

How well did you know this?
1
Not at all
2
3
4
5
Perfectly
38
Q

A woman who had been living in a foreign
country for many years traveled to the United
States to assist her brother, who had been
indicted for a serious crime committed in State
A. She retained a lawyer who resided in State A
to represent her brother, and then she returned to the foreign country. Thereafter, the lawyer sent
the woman a bill for $100,000 for his services,
but she refused to pay. Before she moved to the foreign country, the woman had resided in State B. Although the woman is still a citizen of the United States, she had moved to the foreign country 10 years ago, intending to live there
permanently.May the lawyer commence an action against the woman for breach of contract in the federal court for State A?
(A) Yes, because the action is between citizens
of different states.
(B) Yes, because the action is between a citizen
of a state and an alien.
(C) No, because the federal court for State A
may not exercise personal jurisdiction over
the woman.
(D) No, because the court lacks subject matter
jurisdiction.

A

(D) The court lacks subject matter jurisdiction of the action because the lawyer’s state law claim is
supported neither by diversity of citizenship jurisdiction nor by alienage jurisdiction. Although
the woman is a citizen of the United States, she is not a citizen of any state of the United States.
A person is a citizen of the state or country where she is domiciled. The question tells us that the
woman moved from State B to a foreign country and has been living there for many years. Thus,
it is reasonable to conclude that she changed her domicile from State B to the foreign country. However, the woman is not an alien; rather she is a United States citizen without a state citizenship.

(B) is incorrect because both the lawyer and the woman are citizens of the United States. A
United States citizen may become a citizen of a foreign country by renouncing her United States
citizenship and acquiring the citizenship or nationality of the foreign country by a formal process called naturalization. Here, the woman is not an alien.

(C) is incorrect because a court in State A would have specific personal jurisdiction over the woman because she entered into a contract in
State A with the lawyer (her agreement to retain and pay him) and the lawyer’s claim arises out of
that contract.

How well did you know this?
1
Not at all
2
3
4
5
Perfectly
39
Q

The plaintiff, a citizen of State A, filed suit against the defendant, also a citizen of State A, in federal district court, alleging that the defendant had failed to perform a contract to provide 1,000 fully automatic machine guns. The defendant claimed that a recently enacted federal statute made the manufacture of fully automatic machine guns illegal.

Federal question jx?

A

The court does not have federal question jurisdiction because the recently enacted federal
statute arises only in anticipation of the defendant’s defense. This is insufficient to confer federal question jurisdiction.

How well did you know this?
1
Not at all
2
3
4
5
Perfectly
40
Q

The plaintiff and the defendant are both citizens of State A. The plaintiff wished to sue the defendant in a federal court on a $100,000 claim. For this reason, and only this reason, she moved permanently to State B and filed suit in a federal court there, with jurisdiction being based on diversity of citizenship. A few days after being properly served, the defendant, after accepting a job offer from a company based in State B, moved permanently to State B. For purposes of the plaintiff’s case, what are the citizenships of the parties?

(A) Both the plaintiff and the defendant are
citizens of State B.
(B) The defendant is a citizen of State A, and
the plaintiff is a citizen of State B.
(C) The defendant is a citizen of State B, and
the plaintiff is a citizen of State A.
(D) Both the plaintiff and the defendant are
citizens of State A.

A

(B) The defendant is a citizen of State A, and the plaintiff is a citizen of State B.

Diversity is determined at the time the action is filed, not when the cause of action accrues or after the action commences. Also, the plaintiff’s motive for moving to State B is irrelevant, as long as the change of citizenship is genuine; i.e., she intends to remain there. The defendant’s motive is also irrelevant.
Even though he moved to State B for reasons unrelated to the case and his move appears to
be enough to change his citizenship, citizenship is still determined at the time the action is filed,
and thus he is still considered to be a citizen of State A for purposes of this lawsuit.

How well did you know this?
1
Not at all
2
3
4
5
Perfectly
41
Q

Three years ago a woman’s husband abandoned her and their home in State A and moved to State B, where he is now a citizen. The woman now files against her husband for divorce in federal court in State A, seeking a property settlement valued at $200,000. The husband moved to dismiss for lack of jurisdiction. Will the court grant the husband’s motion?

(A) No, because diversity of citizenship is present
and the amount in controversy exceeds
$75,000.
(B) No, if State A has a long arm statute
granting personal jurisdiction over the
husband.
(C) Yes, if the husband moves to State A after
suit is filed.
(D) Yes, because federal courts do not have
subject matter jurisdiction over domestic
relations cases.

A

(D) The court will grant the husband’s motion.

Even if diversity and the minimum amount in controversy are present, the federal district courts will not exercise jurisdiction over domestic relations or probate proceedings.

How well did you know this?
1
Not at all
2
3
4
5
Perfectly
42
Q

A supplier of rivets contracted to supply rivets to an aircraft manufacturer. The aircraft manufacturer cancelled the order, and the supplier intends to file a breach of contract action against the aircraft manufacturer, seeking damages of $600,000. The supplier is a State A corporation, and all its operations are in State A. The aircraft manufacturer is a State B corporation.
Its corporate headquarters are in State C, but its manufacturing operations are in State A. The supply contract was signed in State A, and it provided that the rivets would be delivered and
paid for in State A. Can the supplier maintain an action based on diversity of citizenship jurisdiction in a federal district court?

(A) No, because both corporations are citizens
of State A.
(B) No, because all relevant events giving rise
to the claim occurred in one state.
(C) Yes, because the supplier is a citizen of
State A and the aircraft manufacturer is a
citizen of State B and State C.
(D) Yes, because the supplier is a citizen of
State A and the aircraft manufacturer is a
citizen only of State B.

A

(C) The supplier can maintain the action. Diversity of citizenship jurisdiction requires complete diversity— meaning that no plaintiff may share state citizenship with any defendant—and the amount in controversy must exceed $75,000. Here, the issue is the citizenship(s) of the parties. For purposes of diversity of citizenship jurisdiction, a corporation is deemed to be a citizen of every state in which it is incorporated and the one state in which it has its principal place of business. The supplier is a citizen of State A only, given that State A is both its state of incorporation and its principal place of business. The aircraft manufacturer is a citizen of State B, its state of incorporation, and the state identified as its principal place of business. The United States Supreme Court has held that the principal place of business is the corporate headquarters where the corporation’s high level officers direct and control the corporation’s activities. Here, the aircraft manufacturer’s headquarters are in State C, making State C its principal place of business despite the fact that its manufacturing operations are in State A.

(A) is not correct because, as explained above, a corporation’s principal place of business is the state from which the corporation’s high level officers direct and control the corporation’s activities, not the state in which it has all of its physical operations.
(B) is not correct because the place where the events took place has no role in diversity of
citizenship jurisdiction. (The events will determine where venue would be proper, but that is a separate question from subject matter jurisdiction.)

(D) is not correct because a corporation may
have two (or more) state citizenships, as explained above.
How well did you know this?
1
Not at all
2
3
4
5
Perfectly
43
Q

An employee sued his employer in federal
court in State B, alleging that the employer
violated federal civil rights statutes by permitting
a hostile work environment in which the employee was afraid to go to work due to harassment from a co-worker. The co-worker is from State A, and both the employer and employee are from State B. The employee properly joined the co-worker under the permissive party joinder rule. The law in State B provided the basis for the employee’s claim against the co-worker, and neither claim exceeded $20,000. Does the federal court have supplemental jurisdiction over the employee’s claim against the co-worker?

(A) Yes, because supplemental jurisdiction exists
whenever the primary claim is based on
a federal question.
(B) Yes, because supplemental jurisdiction may
include claims that involve the joinder or
intervention of additional parties.

A

(B) The court has supplemental jurisdiction.

When jurisdiction is based on a “federal question” claim, and diversity jurisdiction is not available, a nonfederal claim can be joined only if the court has supplemental (pendent) jurisdiction over it. The court will have supplemental (pendent) jurisdiction over the claim if it arose from the same transaction or occurrence as the federal claim.
Here, although diversity jurisdiction is not available in the claim between the employee and the co-worker because the amount in controversy does not meet the more than $75,000 jurisdictional limit, the court will have supplemental jurisdiction because the hostile work environment claim arose out of the same transaction or occurrence as the federal civil rights claim.

(A) is wrong because it is an incomplete statement of the law. While supplemental jurisdiction may exist whenever the original claim is based on a federal question, to permit it, the state law claim also must be part of the same transaction or occurrence.

How well did you know this?
1
Not at all
2
3
4
5
Perfectly
44
Q

The court may decline to exercise its supplemental
jurisdiction on one of four bases:

A

(i) the case presents a novel or complex issue of state law;
(ii) in a federal question case, the nonfederal claim substantially predominates the case;
(iii) the claims over which the federal court had original jurisdiction have been dismissed (taking into consideration the amount of time the court has spent on the case); or

(iv) there are extraordinary circumstances
for declining jurisdiction.

How well did you know this?
1
Not at all
2
3
4
5
Perfectly
45
Q

A State A plaintiff filed a breach of contract action against a State B defendant in federal district court, seeking $100,000 in compensatory damages. Three months after the plaintiff filed and served the complaint, the plaintiff and defendant were in an automobile accident. May the State A plaintiff assert a negligence claim against the State B defendant in the pending breach of contract action to recover damages incurred in the automobile accident?

(A) Yes, but only if the negligence claim seeks
more than $75,000.
(B) Yes, regardless of the amount of the negligence claim, because the court on motion
may permit a plaintiff to file a supplemental
pleading setting out events that happened
after the pleading to be supplemented.
(C) No, because the two claims do not arise
from the same transaction or occurrence or
series of transactions or occurrences.

(D) No, because the time for filing an amended
or supplemental complaint has expired.

A

(B) The plaintiff may assert the negligence claim if permitted by the court.

The court on motion may permit a plaintiff to file a supplemental pleading that relates to matters occurring after the date of the original pleading. [Fed. R. Civ. P. 15(d)]

(A) is incorrect because a plaintiff may aggregate
all his claims against a single defendant to meet the jurisdictional amount, regardless of whether
the claims are legally or factually related. Since the contract action already exceeds $75,000, the
amount in controversy in the negligence action is irrelevant.

(C) is incorrect because a plaintiff may bring all claims he has against a defendant in a single action regardless of whether they are related.

(D) is incorrect because there is no time limit on when a supplemental pleading can be filed.

How well did you know this?
1
Not at all
2
3
4
5
Perfectly
46
Q

Objections to service of process must be asserted in …

Objections to venue must be asserted in…

A

… the defendant’s first response to the complaint—whether that response is a Rule 12(b) pre-answer motion or an answer—or else it is waived.

… the defendant’s first Rule 12 response to the complaint—whether that response is a pre-answer motion or an answer.

How well did you know this?
1
Not at all
2
3
4
5
Perfectly
47
Q

A party may amend a responsive pleading of
right within …

A

… 21 days after serving it.

Thereafter, the party may amend only with consent of all parties or with leave of the court, but the “court should freely grant leave when justice so requires.”

How well did you know this?
1
Not at all
2
3
4
5
Perfectly
48
Q

A State A citizen filed an antitrust action against an State B corporation in federal district court. Fourteen days after filing the complaint and before the State B corporation filed any response, the State A citizen sought to file an
amended complaint, adding a breach of contract claim against the same State B corporation for
more than $75,000. The breach of contract claim
was completely unrelated to the antitrust claim,
and the facts underlying the two claims were completely different and distinct. May the State A plaintiff amend his complaint?

(A) No, because the amendment involves a
claim that does not arise from the same
transaction or occurrence as the original
pleading.
(B) No, because a plaintiff may not join
unrelated claims in the same action.
(C) Yes, but only with leave of the court or
written consent of the defendant.
(D) Yes, even without leave of the court or
written consent of the defendant.

A

(D) The plaintiff may amend his complaint.

A party may amend a pleading to which a responsive pleading is required of right if the party does so within 21 days after the responsive pleading or other response is served. [Fed. R. Civ. P. 15(a)(1)(B)]

(A) is incorrect because the amendments to
a complaint do not have to arise from the same transaction or occurrence as the original pleadings.
(B) is incorrect because a plaintiff may bring all claims he has against a defendant regardless
of whether they are related.

(C) is incorrect because a complaint may be amended once as a matter of course within 21 days of serving it or 21 days after service of a responsive pleading or pre-answer motion. Thereafter, a pleading may be amended only by the written consent of the defendant or by leave of the court.

How well did you know this?
1
Not at all
2
3
4
5
Perfectly
49
Q

Parties may join as plaintiffs or be joined as
defendants whenever …

A

(i) some claim is made by each plaintiff and against each defendant relating to or arising out of the same series of occurrences or transactions; and

(ii) there is a question of
fact or law common to all parties.

How well did you know this?
1
Not at all
2
3
4
5
Perfectly
50
Q

An employee sued her employer in federal
court alleging that she was discriminated against
on the basis of her gender in violation of federal
law. At the close of the employee’s evidence, the
employer moved for judgment as a matter of law,
contending that the employee failed to submit
adequate evidence to prove her case. At the close of all evidence, neither party made any motions, and the case was submitted to the jury. The jury returned a verdict for the employee, finding that she was discriminated against on the basis of gender. Twenty-five days after the judgment was filed, the employer filed a renewed motion for judgment as a matter of law.
Should the court grant the motion?

(A) No, because the employer did not move for
judgment as a matter of law at the close of
all the evidence.

(D) Yes, if the court finds that the verdict is
against the weight of the evidence.

A

(C) The court should grant the employer’s motion if it believes a jury would not have had a legally
sufficient basis to find for the jury.

If a defendant moves for judgment as a matter of law under Rule 50(a) at any point during the trial, it may renew that motion after the jury reaches a verdict, regardless of whether it had moved for judgment as a matter of law at the close of all the evidence.
The motion to renew must be made within 28 days after the jury returns its verdict.

(D) is incorrect because it gives the wrong standard. A court may only grant a motion for judgment as a matter of law if there is no legally sufficient evidentiary basis for it. In deciding the motion, the trial judge is not free to weigh the evidence. Rather, she must view it in the light most favorable to the verdict winner.

How well did you know this?
1
Not at all
2
3
4
5
Perfectly
51
Q

A plaintiff filed a proper diversity suit in federal district court against a defendant for battery stemming from an altercation the two had in a bar. The two parties were former business partners, and the defendant filed a counterclaim against the plaintiff for breach of contract relating to their former business dealings. The plaintiff filed a motion for summary judgment in the battery suit, and the court granted it. The court’s order simply stated: “Plaintiff’s motion for summary judgment on the battery claim is granted.” The defendant appeals the grant of summary judgment. Can the court of appeals hear the defendant’s appeal?

(A) Yes, because the summary judgment ruling
was a final judgment on the merits.

(D) No, because there has not been a final
judgment on the merits since the contract
claim has not been adjudicated

A

(D) The court of appeals cannot hear the defendant’s appeal because there has not been a final judgment on the merits in this case.

With the exception of some interlocutory orders, only final orders are reviewable on appeal. _A final order on the merits disposes of the whole case
by rendering judgment as to all the parties and all causes of action._When multiple claims or
parties are involved in an action, the court may enter a “final judgment” as to fewer than all of
the claims or parties only on an express determination that there is no just reason for delay. If the court does not make such an express determination, a judgment as to fewer than all the claims or parties is not a final judgment and is not appealable.

Here, the court has not made an express determination that there is no just reason for delay. Therefore, the summary judgment ruling is not appealable until there is also a final judgment in the contract action.

(A) is incorrect because a summary judgment ruling can be a final judgment on the merits, but there has not been a final judgment here because of the outstanding contract dispute.

How well did you know this?
1
Not at all
2
3
4
5
Perfectly
52
Q

A motorcyclist, a car driver, and a truck driver were involved in a three-vehicle accident in a busy intersection. The motorcyclist filed a negligence action against the car driver for personal injuries suffered in the accident. In the
car driver’s pleadings, she denied that she was negligent and raised the motorcyclist’s contributory negligence as a defense. The jury returned a general verdict in favor of the car driver. The motorcyclist then filed a negligence action against the truck driver. At the appropriate
time, the truck driver filed a motion to dismiss
for failure to state a claim, asserting that the
motorcyclist was precluded from re-litigating his
contributory negligence. Is the truck driver likely to be successful?
(A) Yes, because there was a final judgment on
the merits in the first case.
(B) Yes, because the jury in the first case determined
that the motorcyclist was contributorily
negligent.
(C) No, because it is not clear whether the jury
in the first case found that the motorcyclist
was negligent.
(D) No, because only someone who was a party
in the first action can assert issue preclusion
in the second.

A

(C) The truck driver is unlikely to be successful because it is unclear whether the jury in the first case found that the motorcyclist was contributorily negligent.

For issue preclusion to apply to an issue,
the issue must actually have been litigated and determined in the previous case.

Here, the issue of the motorcyclist’s negligence was not determined in the first case. The jury could have found that the motorcyclist was contributorily negligent, or it simply could have found that the motorcyclist did not prove that the car driver was negligent. Because this issue was not decided, the truck driver cannot assert issue preclusion (collateral estoppel) against the motorcyclist. (B) is therefore incorrect.

(A) is incorrect because issue preclusion requires more than a final judgment on the merits in the first case. Issue preclusion applies to issues that were actually litigated and essential to the judgment in the first case and are relevant in the second case.

(D) is incorrect because nonparties may be allowed to assert issue preclusion and use a prior judgment to avoid liability in a second suit.

How well did you know this?
1
Not at all
2
3
4
5
Perfectly
53
Q

An airplane passenger sued an airplane
manufacturer in federal court after the passenger
was injured in a plane crash. The passenger alleged that a defective part made by the
manufacturer had caused the crash. The case
went to trial, and the jury found in favor of the
passenger. Judgment was entered on March 1.
Six months later, an airplane parts subcontractor
announced a recall of an engine part that had
been found to be defective and was the likely
cause of two previous plane crashes. Until this
announcement there had been no indication to
the public or aviation industry that this part was
defective, because the subcontractor had been
very secretive about its in-house investigation.
This engine part had also been in the passenger’s plane. In light of this new evidence, the manufacturer files a motion for relief from a
final judgment. Is the court likely to grant the manufacturer’s motion?
(A) Yes, because there is newly discovered
evidence that by due diligence could not
have been discovered in time to move for a
new trial.
(B) Yes, because the original judgment is now
void.
(C) No, because motions for relief from a final
judgment must be brought within 28 days
of the entry of judgment.
(D) No, because the manufacturer should have
filed a motion to amend a final order.

A

(A) The court is likely to grant the manufacturer’s motion.

A court may relieve a party from a final
judgment or order on the ground of newly discovered evidence that by due diligence could not have been discovered in time to move for a new trial.

A motion for a new trial must be filed no
later than 28 days after judgment is entered.

Here, the defendant discovered the new evidence about the recalled engine part six months after the judgment was entered; therefore, it was too late for a motion for new trial.

(B) is incorrect because the original judgment is not void. A judgment is void only if there is a fundamental flaw, such as lack of jurisdiction or due process.

(C) is incorrect because motions for relief from a final judgment on the basis of newly discovered evidence must be brought within one year.

(D) is incorrect because, although a party may file a motion to amend a final order on the basis of newly discovered evidence, it must be filed within 28 days of the order. Because six months have passed since the judgment was entered, this motion is not available to the manufacturer.

How well did you know this?
1
Not at all
2
3
4
5
Perfectly
54
Q

In order to reserve its right to appeal an error
in an instruction given or a failure to give an instruction, a party must …

A

… object on the record before the instructions are given.

If adequately preserved, the instructions are reviewed using an abuse of discretion standard.

However, if the objection is not made (and therefore not preserved for full appellate review), the court’s review is limited to considering whether there was a plain error in the instruction that affected substantial rights. Relief based on plain error is difficult to obtain.

How well did you know this?
1
Not at all
2
3
4
5
Perfectly
55
Q

The plaintiff sued the defendant in federal court to recover damages after the defendant crashed his car into the plaintiff’s garage. Judgment was entered for the plaintiff in a valid, final judgment on the merits. The plaintiff had originally thought that the damage done to his yard was minimal, but he later discovered that the yard’s elaborate landscaping would have to be replaced at a high cost. He then files suit against the defendant to recover damages for the property damage to his yard. Is the plaintiff’s claim likely to be successful?

(B) Yes, because the plaintiff did not realize the
extent of the damage to his yard until after
the conclusion of the first case.
(C) No, because the suit is barred by issue
preclusion.
(D) No, because the plaintiff is barred by
claim preclusion from bringing the second
lawsuit.

A

(D) The plaintiff’s claim is not likely to be successful because it is barred by claim preclusion principles.

Claim preclusion requires that:

(i) a valid, final judgment on the merits was entered in the first case;
(ii) the cases were brought by the same claimant against the same defendant; and
(iii) the same cause of action is involved in the later lawsuit.

Generally, a claimant is required to assert all causes of action arising out of the same transaction or occurrence that is the subject matter of the claim.

Here, the first case was resolved by a valid, final judgment on the merits. Both cases involve the same claimant, same defendant, and claims arising from the same transaction or occurrence. The plaintiff should have sought damages for his yard in the first lawsuit because the second lawsuit is barred by claim preclusion.

(B) is incorrect because it is irrelevant when the
plaintiff realized the extent of the damage to his yard.

(C) is incorrect because issue preclusion
involves binding a party in a different cause of action as to certain issues that were litigated in the first action. Since the same cause of action is involved here, claim preclusion is appropriate.

How well did you know this?
1
Not at all
2
3
4
5
Perfectly
56
Q

A patient properly sued her doctor for
medical malpractice in federal court. Neither
party requested a jury trial. Following witness
testimony from both sides, the judge was not
convinced the doctor breached the applicable
standard of care. Over the plaintiff’s objection
that she should be allowed to present
evidence on proximate causation and damages,
and without hearing any evidence on proximate
causation or damages, the judge entered
judgment for the doctor, stating his findings of
fact and conclusions of law orally on the record.
Has the judge committed reversible error?

(B) Yes, because the judge did not state his
findings of fact and conclusions of law in
writing on the record.
(C) No, because the judge’s findings of fact and
conclusions of law may not be disturbed on
appeal.
(D) No, because findings of fact and conclusions
of law may be orally stated on the
record.

A

(D) The judge did not commit reversible error.

In a case tried before a judge, once a party has been fully heard on an issue of fact, the judge may enter judgment on partial findings where that issue would dispose of the case.

Here, the patient’s inability to prove that the doctor breached the applicable standard of care would be fatal to her case, and thus the judge did not err when he entered judgment once that became clear. (Note that a judge may continue hearing evidence; whether to do so is a discretionary call.) Thus, (D) is correct and (A) is incorrect.

(B) is incorrect because the judge may issue findings of fact and conclusions of law orally on the record.

(C ) is an incorrect statement of the law.

An appellate court may overturn the judge’s findings of fact on appeal, although the judge will be given considerable discretion. (A clearly erroneous standard is applied.) Purely legal aspects of the case are reviewed de novo.

How well did you know this?
1
Not at all
2
3
4
5
Perfectly
57
Q

A plaintiff filed a tort action against a defendant
in federal district court. The defendant filed
no pre-answer motions and proceeded to file
and serve his answer, responding to the merits
of the plaintiff’s complaint. Three months later,
after discovery had begun, the defendant filed a
motion for judgment on the pleadings, asserting
as defenses lack of personal jurisdiction, lack of
subject matter jurisdiction, and failure to state a
claim upon which relief may be granted.
Which of these three defenses, if any, were
timely raised?
(A) Only the subject matter jurisdiction defense
was timely raised, because the defendant
waived the personal jurisdiction and failure
to state a claim defenses by not asserting
them in his answer.
(B) Only the defenses of subject matter jurisdiction and personal jurisdiction were timely
raised, because the defendant waived the
defense of the plaintiff’s failure to state a
claim by not asserting that defense in his
answer.
(C) Only the defenses of subject matter jurisdiction and failure to state a claim were
timely raised, because the defendant waived
the personal jurisdiction defense by not
asserting that defense in his answer.
(D) All three of these defenses were timely
raised.

A

(C) The defenses of subject matter jurisdiction and failure to state a claim were timely asserted.

The lack of subject matter jurisdiction may be raised at any time.

The defense of failure to state a claim upon which relief can be granted can be asserted in

a pre-answer motion,

any pleading,

a motion for judgment on the pleadings, or

at trial.

Because the defendant asserted this defense in a
motion for judgment on the pleadings, it was timely asserted. Hence, (A) and (B) are incorrect.

A defendant must object to lack of personal jurisdiction in a pre-answer motion or the answer. If the defendant does not, the objection is waived.

Because the defendant here did not assert the defense of lack of personal jurisdiction in his answer or any pre-answer motion, he waived the defense. Therefore, the personal jurisdiction defense was not timely asserted. Hence, (B) and (D) are incorrect.

How well did you know this?
1
Not at all
2
3
4
5
Perfectly
58
Q

An investor and a chef agreed to open a
restaurant. They entered into a contract in which
the investor agreed to provide the funding and
the chef agreed to operate the restaurant. When
the investor failed to provide the agreed funding,
the chef filed a breach of contract action against
the investor in a federal district court, seeking
$150,000 in damages. The investor is a citizen
of State A. The chef is a citizen of a foreign
country, but he was admitted into the United
States for permanent residency and is now
domiciled in State A.
Does the federal court have subject matter
jurisdiction over the action?
(A) Yes, because the claim is between a citizen
of a state and a citizen of a foreign country
and the amount in controversy exceeds
$75,000.
(B) Yes, because the action involves an international
transaction.
(C) No, because federal courts do not have
jurisdiction over claims asserted by citizens
of foreign countries unless the claim arises
under federal law.
(D) No, because jurisdiction is withdrawn when
the action is between citizens of a U.S.
state and foreign citizens who are lawfully
admitted for permanent residence in the
U.S. and are domiciled in the same state.

A

(D) The court does not have subject matter jurisdiction. Provided the amount in controversy exceeds $75,000, the diversity statute authorizes jurisdiction over a civil action between citizens of a U.S. state and citizens of a foreign state, except that jurisdiction is withdrawn if the action is between citizens of a U.S. state and citizens of a foreign state who are lawfully admitted for permanent residence in the United States and are domiciled in the same U.S. state.

Here, the investor is a citizen of State A. The chef has been admitted to the United States for permanent residence, and he is domiciled in State A. Because the investor and chef share domicile in State A, jurisdiction is withdrawn.

How well did you know this?
1
Not at all
2
3
4
5
Perfectly
59
Q

A pedestrian was injured in a car accident
involving two cars. The pedestrian filed a negligence action in federal district court against the irst driver, seeking $100,000 in damages. The
pedestrian is a citizen of State A and the first
driver is a citizen of State B. The first driver
then filed a third-party claim against the second
driver, claiming that the second driver is responsible for half of the harm caused to the pedestrian and seeking to recover half of any liability the first driver is found to have to the pedestrian. The second driver is a citizen of State A. Does the federal court have subject matter
jurisdiction over the third-party claim asserted
by the first driver against the second driver?

(A) Yes, because the court has supplemental
jurisdiction over the third-party claim.

(C) No, because the amount in controversy in
the third-party claim is too small.

A

(A) The court has supplemental jurisdiction over the third-party claim.

Diversity of citizenship jurisdiction is available when (i) there is complete diversity of citizenship, meaning that each plaintiff must be a citizen of a different state from every defendant; and (ii) the amount in controversy exceeds $75,000. In the instant case, the case is properly in federal court because diversity jurisdiction exists for the underlying claim (i.e., the claim by the pedestrian against the first driver), given that the pedestrian is from State A, the first driver is from State B, and the amount in controversy is $100,000.

The third-party indemnity claim, however, cannot invoke diversity jurisdiction, even though the first driver is from State B and the second driver is from State A, because the amount claimed is $50,000.

When the federal court has subject matter jurisdiction over one claim, it has discretion to exercise supplemental jurisdiction over related claims that derive from the same common nucleus of fact and are such that a plaintiff would ordinarily be expected to try them in a single judicial proceeding. (Essentially,
this means that the supplemental claim must arise from the same common nucleus of operative fact as the claim invoking federal subject matter jurisdiction.) In the instant case, the claims of pedestrian (the underlying claim) vs. first driver and of first driver vs. second driver (the indemnity claim) are derived from the same accident, and thus both are derived from the same common nucleus of operative fact.

There are restrictions on the use of supplemental jurisdiction when the use of supplemental jurisdiction would be contrary to diversity jurisdiction. In terms of third-party
practice, claims by a plaintiff against an impleaded party may not use supplemental jurisdiction to circumvent the diversity statute. However, claims by a defendant are not listed among the restrictions; thus, supplemental jurisdiction is available to a defendant (third-party plaintiff) against a third-party defendant. Therefore, the first driver may use supplemental jurisdiction to have his claim against the second driver heard in federal court. (This would be true even if they were from the same state, so long as the claim was a true indemnity claim.) As a result, (A) is correct, and (C) incorrect.

How well did you know this?
1
Not at all
2
3
4
5
Perfectly
60
Q

A company refused to hire a woman based
on her religious beliefs in violation of federal
civil rights laws. The company was incorporated
in State A, and its principal place of business was in State B. The events relating to the woman’s claim occurred in State B. Thereafter, the woman found employment in State A. She moved to State A and commenced an action against the company in the federal court for State A. She hired a process server to serve the
company, who served the company’s treasurer at
its offices in State B. State A and B each authorize service of process on corporations only by personal delivery of the summons and complaint on its president, vice president, or secretary. The company has timely moved to dismiss the complaint based on improper venue and improper service of process.
How should the court decide the motion?

(B) The court should grant the motion because
service of process is improper.

(D) The court should deny the motion.

A

(D) Service on the treasurer was effective service.

Under the Federal Rules, a corporation may be
served by serving any corporate officer or managing or general agent. Alternatively, service may be made under state rules or by mail under the waiver of service provision.

Here, service is proper because the treasurer, an officer of the corporation, was served. Service need not comply with state law as well. Service in accordance with state law is an authorized alternative to the service methods specified in the Rules.

Venue is proper because the corporation resides there. Venue is proper in

(i) a judicial district in which any defendant resides, if all defendants are residents of the state in which the district is located;
(ii) a judicial district in which a substantial part of the events or omissions giving rise to the claim occurred or in which a substantial part of property at issue is located; or
(iii) if there is no district anywhere in the United States which satisfies (i) or (ii), a judicial district in which any defendant is subject to the court’s personal jurisdiction with respect to such action.

A corporate defendant resides in any federal judicial district in which it is subject to the court’s personal jurisdiction with respect to the civil action in question.

Here, for venue purposes, the company resides in the federal district of State A because it would be subject to that district court’s personal jurisdiction in this action. Specifically, it would be subject to the court’s general personal jurisdiction because it was incorporated in State A, meaning it could be sued in State A on any claim, including the civil action in question, which arose from acts in State B.

How well did you know this?
1
Not at all
2
3
4
5
Perfectly
61
Q

Three drivers were involved in a three-car
accident in a city in the Middle District of State
A. One of the drivers was severely injured and
intends, in good faith, to file a negligence action
seeking $500,000 in damages against the other
two in federal court. The plaintiff is a citizen of
State A and resides in the Middle District. One
defendant is a citizen of State B and resides in
its Northern District. The other defendant is a
citizen and resident of State C, a single district
state. At the time of the accident, the State C
defendant was staying in a State A hotel for
two months while consulting on a construction
project.
In what district or districts is venue proper?
(A) The Middle District of State A only.
(B) The Northern District of State B and the
District of State C only.
(C) The Middle District of State A, the
Northern District of State B, and the
District of State C.
(D) None; the action may not be maintained in
federal court because federal subject matter
jurisdiction does not exist.

A

(A) Venue is proper in the Middle District of State A only.

Federal venue in civil actions is proper in
(i) the district where any defendant resides, if all defendants are residents of the state in which the
district is located; and

(ii) the district in which a substantial part of the events or omissions giving rise to the claim occurred.

Here, the defendants do not reside in the same state. Therefore, venue cannot be based on the residence of the defendants. However, the accident occurred in the Middle District of State A, which is a proper venue under prong (ii).

How well did you know this?
1
Not at all
2
3
4
5
Perfectly
62
Q

A citizen of State A filed a complaint alleging
negligence by two parties, both of whom reside
in State B. The complaint was filed in the United
States District Court of State B. However, the
accident took place in State D, the plaintiff was
treated by an emergency room physician in
State D, and all of the witnesses reside in State
D. Two months after they filed their respective
answers, which did not address any problems
with personal jurisdiction or venue, the defendants
filed a joint motion to transfer the case to
the District of State D.
May the court grant the motion?

(A) No, because the defendants have waived
any issue as to venue by not raising the issue
of venue in the answer.

(D) Yes, because transferring the case to the
District of State D could be “in the interests
of justice.”

A

(D) The court could grant the motion.

Pursuant to Rule 12(b), improper venue must be raised in a defendant’s first response—either in its timely motion to dismiss before the answer or in the answer, whichever is first.

Here, though, venue was initially proper because the defendants resided in State B. Rather, the issue is whether the court may transfer the case from one proper venue to another. Such a transfer has no strict time limit. Rather, transfer is left to the discretion of the trial judge, and the judge may refuse transfer where the case has been pending for some time and would work a prejudice to one of the parties. Two months is likely not a sufficient period to prevent transfer of the case. Thus, (D) is correct.

(A) is incorrect because, as explained, transfer of venue in the interests of justice does not fall under the “first pleading or motion” rule.

How well did you know this?
1
Not at all
2
3
4
5
Perfectly
63
Q

A homeowner from State A hired a contractor
from State B to build a vacation home for her in
State C. The parties signed the contract in State
A. The contractor breached the contract, and
the homeowner sued the contractor in a court
of State A, seeking damages of $100,000. The
contractor removed the case to the federal court
for State A. The homeowner promptly moved
to remand the case to state court, arguing that
venue was improper.
Which of the following facts is most relevant
to the court’s decision on the homeowner’s
motion?
(A) The contract was signed in State A.
(B) The contractor resides in State B.
(C) The homeowner commenced the action in a
State A court.
(D) The vacation home was to be built in State
C.

A

(C) The commencement of the action in State A is most relevant.

When a case is removed from state court to federal court, venue is set in the federal district court that embraces the state court in which the action was pending, making the federal district court of State A the only appropriate venue choice. Thus, (C) is the correct answer because the most (indeed, only) relevant fact is the
fact that the action was commenced in a court of State A. Unlike cases commenced in federal
court, in cases removed to federal court, the residence of the defendant contractor (answer choice (B)) and the place where a substantial part of events relating to the claim occurred (answer
choices (A) and (D)) are irrelevant.

How well did you know this?
1
Not at all
2
3
4
5
Perfectly
64
Q

A State A citizen and a State B citizen were
in a car accident in State C. The State A citizen
filed a negligence action against the State B
citizen in a State C state court, seeking $500,000
in damages. If the State B citizen wishes to remove the action to federal district court, in which federal district should the State B citizen file a notice of removal?

(A) In either the district in State C in which the
accident took place or the district in State B
in which the State B citizen resides.

(C) In only the district in State C in which the
State C state court is located.

A

(C) The notice of removal should be filed in the district in State C in which the State C court is
located.

The federal removal statute provides that

_the notice of removal should be filed in the
federal district court for the district that geographically encompasses the state court from which the action is being removed._Thereafter, copies must be sent to the parties and filed with the applicable state court.

Thus, (A), (B), and (D) are incorrect. (A) describes the general venue rule—i.e., an action may be filed in the judicial district in which any defendant resides (if they are all from the same state) or in which a substantial part of the events took place—for actions filed originally in federal court.

How well did you know this?
1
Not at all
2
3
4
5
Perfectly
65
Q

While making deliveries with a company van,
an employee of a bakery collided with a bicycle.
To recover for his extensive injuries, the cyclist
sued the employee in the appropriate federal
court. The court granted summary judgment in
favor of the cyclist, finding that the employee’s
negligence was the sole cause of the collision.
Thereafter, the cyclist died. The bakery commenced an action against the cyclist’s estate
for damage to the van. The cyclist’s estate moved
to dismiss the bakery’s complaint, based on the
finding of negligence in the prior action. If the court denies the motion, what is the most likely reason?
(A) The bakery was not a party to the prior action.
(B) The cyclist’s estate was not a party to the
first action.
(C) The cyclist’s action against the employee
was not fully litigated.

A

(A) Generally, a party may assert issue preclusion offensively if

(i) the prior action ended in a final judgment;

(ii) the issue has been actually litigated and determined;

(iii) the issue was essential to the judgment;

(iv) the party to be bound by the prior judgment was a party to the prior action or in privity with a party to the prior action; and

(v) the party asserting issue preclusion was
a party to (or in privity with a party) to the prior action or, if not, is asserting issue preclusion
under circumstances that are not unfair or inequitable.

Here, the fourth element is missing. The
bakery was not a party to the prior action and therefore the cyclist’s estate may not assert issue
preclusion against it.

(B) is incorrect because the cyclist and the cyclist’s estate are in privity. Therefore, issue preclusion may be asserted by it.

(C) is incorrect because a case is fully litigated
when it is resolved by the grant of a motion for summary judgment.

How well did you know this?
1
Not at all
2
3
4
5
Perfectly
66
Q

A company manufactured and sold a product called “True Glue.” An inventor brought an action in federal court against the company, alleging that the product infringed a patent owned by him. The company denied the allegations of the inventor’s complaint. In addition, it asserted a counterclaim against the inventor for breach of contract, based on a transaction between them unrelated to the inventor’s claim.
After the completion of discovery, the company
moved for summary judgment dismissing the
inventor’s claim. The court granted the motion,
thus leaving only the company’s counterclaim for
breach of contract to be adjudicated.
Which of the following statements most
accurately describes the inventor’s right to
appeal the court’s grant of the motion for
summary judgment?
(A) The inventor may immediately appeal the
judgment because summary judgment is a
final judgment.
(D) The inventor may not immediately appeal
the judgment unless the court provides that
it is a final judgment and expressly determines
that there is no just reason for delay.

A

(D) _When a court grants summary judgment on some but not all of the claims in an action, the
court’s order is not final and thus not appealable unless the court expressly determines that there is no just reason to delay entry of judgment.
_Unless the trial judge makes that express determination, its order determining the merits of fewer than all of the claims is not a final judgment and is not appealable. For these reasons,

How well did you know this?
1
Not at all
2
3
4
5
Perfectly
67
Q

A woman sued her employer for sexual harassment. At the close of the trial, the
employer made a motion for judgment as a
matter of law, arguing that the woman’s evidence
was insufficient to establish the elements of her
claim. The court denied the motion.
When the jury returned a verdict in favor of
the woman, the employer renewed its motion
for judgment as a matter of law. In addition to
the renewed motion for judgment as a matter of
law, the employer also moved for a new trial,
asserting that the verdict was against the weight
of the evidence. The court denied both motions.
If the employer appeals the denials of both
the renewed motion for a judgment as a matter
of law and the motion for a new trial, what is the
appropriate standard of review?
(A) De novo for the renewed motion for judgment
as a matter of law and abuse of discretion
for the new trial motion.
(B) Clearly erroneous for the renewed motion
for judgment as a matter of law and de novo
for the new trial motion.
(C) Abuse of discretion for both.
(D) De novo for both.

A

(A) When an appellate court reviews a trial court’s ruling on a post-trial motion for judgment as a matter of law (including a renewed one), it employs a de novo standard. It does so because the issue is one of law.

In contrast, when an appellate court reviews a trial court’s denial of a motion for new trial, it employs a more deferential standard, reversing the trial court’s denial only when there is a clear showing of an abuse of discretion.

Note that when a renewed motion for judgment
as a matter of law and a motion for a new trial are made in the alternative and the renewed motion
is granted, the court rules hypothetically on the new trial motion so that no remand is required if
the ruling on the judgment as a matter of law is subsequently reversed on appeal.

How well did you know this?
1
Not at all
2
3
4
5
Perfectly
68
Q

Jury instructions - obejction

A

To reserve its right to appeal an error in an instruction given or, a failure to give an instruction, a party must object on the record before the instructions are given.

If adequately preserved, the instructions are reviewed using an abuse of discretion standard.

However, if the objection is not made (and therefore not preserved for full appellate review), then the court’s review is limited to considering whether there was a plain error in the instruction that affected substantial rights.

How well did you know this?
1
Not at all
2
3
4
5
Perfectly
69
Q

A passenger sued a limo driver for an injury
the passenger sustained in an accident in State B, and the passenger had the limo driver promptly served with a summons and complaint. Forty days thereafter, with no answer being filed, the passenger requested and was granted an entry of default by the court clerk, and a date for a hearing for a default judgment was set. Notice of the hearing for the default judgment was sent to the limo driver two weeks prior to the hearing.
Assuming that the hearing for the default
judgment has not yet taken place, what is the
limo driver’s best method to get a hearing on the
merits of the case?

(C) Have the entry of the default set aside for
good cause.
(D) Appear at the hearing for a default
judgment.

A

(C) The limo driver’s best approach is to have the entry of default set aside for good cause.

Although there are no facts to indicate that good cause exists to have the default set aside, having the default vacated before the hearing on the default judgment represents the only method by which the limo driver can get a hearing on the merits of the case. If the limo driver can have the entry of default vacated prior to the hearing for a default judgment, the hearing would not proceed, and the case would proceed as any other case in which no default was entered.

(D) is incorrect. The entry of the default cuts off the defendant’s right to contest the case on the merits. _Although the defendant may appear at the hearing for the default judgment *to contest damages*, he may not contest liability until the entry of default is vacated._

How well did you know this?
1
Not at all
2
3
4
5
Perfectly
70
Q

An employee sued her employer in federal court for sexual harassment. The jury returned a verdict in favor of the employer. Three months after the verdict, the employee’s attorney received an anonymous letter stating that a key document presented at trial by the employer had been altered. The employee moved for relief from judgment, alleging that a document presented by her employer at trial had been altered.
Assuming that the employee can show that the
alteration was intentional, how should the court
rule?

(C) Grant the employee’s motion, because fraud and misconduct of an adverse party are proper grounds for relief, and she filed within the appropriate time frame.
(D) Grant the employee’s motion, because fraud
and misconduct of an adverse party are proper grounds for relief, and there is no time limit on such actions.

A

(C) The court should grant the employee’s motion.

A court may relieve a party from a final judgment or order based on fraud, misrepresentation, or other misconduct of an adverse party, _and such a
motion must be made within a reasonable time not to exceed one year.
_

Here, the employee filed her motion based on the adverse party altering a document, which amounts to fraudulent misconduct by the adverse party, and she properly filed her motion within a year. Therefore, the court should grant her motion.

(D) is wrong because there is a one-year time limit to file a motion for relief from judgment based on fraud, misrepresentation, or other misconduct of an adverse party. There is no time limit for such a motion if it is based on a clerical mistake, but that is not the ground that applies to this question.

How well did you know this?
1
Not at all
2
3
4
5
Perfectly
71
Q

If the defendant has not answered or filed a motion for summary judgment, the plaintiff may dismiss her case by filing a notice of dismissal.

A

A plaintiff may voluntarily dismiss an action without prejudice only once, so a second voluntary dismissal operates as an adjudication on the merits.

A dismissal by notice is without prejudice unless the plaintiff has previously dismissed any federal or state court action on the same claim, in which case the dismissal by notice operates as an adjudication on the merits.

How well did you know this?
1
Not at all
2
3
4
5
Perfectly
72
Q

A party can file a motion for a new trial no later
than …

A

… 28 days after judgment was entered.

How well did you know this?
1
Not at all
2
3
4
5
Perfectly
73
Q

An entry of default may be set aside for “good cause shown.”

A

Although not specifically required by the Federal Rules, a majority of courts also will require some showing of a meritorious defense.

How well did you know this?
1
Not at all
2
3
4
5
Perfectly
74
Q

Interlocutory orders are …

A

… the rulings that trial judges make during the course of pretrial proceedings and trials that do not completely resolve the case. Therefore, they are not final. As a result, interlocutory orders are typically not immediately reviewable on appeal until a final order is made, unless they meet one of the exceptions permitting an appeal as of right, i.e.:

  • orders granting injunctions;
  • orders appointing a receiver;
  • orders in admiralty cases finding liability but leaving damages to be assessed later;
  • patent infringement orders where only an accounting is ordered; and
  • orders affecting or changing possession of property.
  • The Interlocutory Appeals Act also may permit a review of an interlocutory order, but it is discretionary, and may be available only when (i) the trial judge certifies that the interlocutory order involves a controlling question of law, as to which there is substantial ground for difference of opinion, and immediate appeal from the order may materially advance the ultimate termination of the litigation; and (ii) the court of appeals then agrees to allow the appeal.
  • A party obtaining such a certificate from the trial judge must, within 10 days, apply to the court of appeals, where two out of three judges must agree to hear the appeal.
How well did you know this?
1
Not at all
2
3
4
5
Perfectly
75
Q

A man invited his partner, who resides in
State B, to his mountain cabin in State A to work
on a business project and enjoy a few days of
relaxation. While hiking on the man’s property,
the partner began to climb over a small fence.
The fence broke in half because the wood had
rotted. The partner fell and suffered a broken
leg, which required hospitalization for several
days.
The partner brought a diversity action in State
A federal court against the man for $100,000 in
damages. The man did not know the fence had
rotted, but the dangerous condition would have
been discoverable applying a reasonable person
standard.
State A follows the traditional common law
rules for landowners and possessors of land.
State B has modified these rules by statute
and applies a reasonable person standard to
dangerous conditions on the land.
At trial, the partner argued that the man
should be liable for failing to exercise reasonable
care to inspect the fence and discover the
dangerous condition. The man contended that
he did not breach his duty of reasonable care
under the State A traditional common law rules
because he did not know the wood in the fence
had rotted. Alternatively, the man argued that the
partner was spending a few days of relaxation as
a social guest, and that he owed neither a duty to
inspect nor to repair unknown dangerous conditions
on his property.
The man moved for a judgment as a matter of
law (“JMOL ”), claiming that, applying State A
law, a reasonable jury would not have a legally
sufficient basis to find for the nonmoving party.
However, the judge denied the motion. The jury
returned a verdict for the partner. In response,
the man filed a motion for a renewed JMOL and,
in the alternative, a motion for a new trial.
What is the proper response to these motions
by the trial judge?
(A) Deny both motions, because the moving
party must elect only one of the two motions
following a jury verdict.
(B) Grant the motion for a renewed JMOL and
exercise judicial discretion to disregard the
motion for a new trial.
(C) Grant the motion for a renewed JMOL ,
but rule hypothetically on the new trial
motion in the event the JMOL is reversed
on appeal.
(D) Set aside the verdict and remand the case
for a redetermination of the applicable law.

A

(C) The proper response by the trial judge is to grant the motion for a renewed JMOL , but rule
hypothetically on the new trial motion in the event the JMOL is reversed on appeal.

A renewed motion for a JMOL is permitted provided the moving party moved for a JMOL at some time during the trial. The moving party is permitted to raise only those issues raised in the initial motion for JMOL.

It is apparent from the facts that the jury did not apply traditional common law rules in arriving at its verdict, which the federal court in State A would be required to follow.
Because of this error, the trial judge should grant the motion for a renewed JMOL. Furthermore, when a renewed motion for a JMOL and a motion for a new trial are made in the alternative, and the renewed motion is granted, the court must rule hypothetically on the new trial motion so that no remand is required if the ruling on the JMOL is subsequently reversed on appeal.

How well did you know this?
1
Not at all
2
3
4
5
Perfectly
76
Q

A bar prep company discovered that its
copyrighted content was being used in an online simulated exam that its competitor was administering in a few days. The company filed
a petition for an ex parte order to direct the competitor to immediately remove that content
from its website. The company submitted an
affidavit specifying why immediate and irreparable injury will result if the exam is administered, and offered to provide security for any costs or damages incurred by the competitor if it was determined that the order was wrongfully
issued. Should the court issue the order?

(A) Yes, because the company submitted an affidavit with specific facts showing immediate
and irreparable injury.
(D) No, because the company has not provided
sufficient certification for obtaining an ex parte order.

A

(D) The court should not issue the ex parte order.

A temporary restraining order may be granted by
a court when it is necessary to prevent irreparable injury to a party, and the injury will result before a preliminary injunction hearing can be held.

As a general rule, notice of the hearing for the issuance of the order must be given before it is issued. However, a court may grant an ex parte temporary restraining order without notice of the hearing to the adverse party if the moving party does the following:

(i) gives specific facts in an affidavit or a verified complaint to establish that immediate and irreparable injury will result to the moving party before the adverse party can be heard in opposition;
(ii) certifies in writing all efforts it made to give notice to the adverse party and why notice should not be required; and
(iii) provides some security to pay for any costs and damages incurred by the adverse party if it is wrongfully enjoined or restrained.

Here, the company alleged irreparable injury and offered to provide security, _but nothing in the
facts indicates that it certified efforts to give notice to the competitor or why notice should not
be required._The facts do not establish that it is impossible to provide notice to the competitor
before a restraining order is issued.

(A) is incorrect because the company’s affidavit is not enough to justify the issuance of an ex parte order, as discussed above.

How well did you know this?
1
Not at all
2
3
4
5
Perfectly
77
Q

A defendant against whom a default is entered loses the right to contest liability. However, …. must still be determined before a default judgment may be entered, and the defaulting party can be heard at the hearing for damages.

If the defendant has “appeared,” even
though he has not answered, he must be notified of the request for a default judgment by ….

A

the amount of damages

… first-class mail at least seven days before the hearing on the application for a default judgment.

Appearance includes any actual formal appearance before the court and any other action that clearly indicates that the defendant intends to contest the case on the merits (e.g., the defendant’s continued settlement negotiations).

Furthermore, an appearance cuts off the clerk’s ability to enter a default judgment.

How well did you know this?
1
Not at all
2
3
4
5
Perfectly
78
Q

An elderly woman entered into a contract with
a company in the business of providing home
care services. Believing that she had been duped
by representatives of the company, the woman
commenced an action in federal court, properly
based on diversity, seeking rescission of the
contract. The company answered, denying the
principal allegations of the woman’s complaint
and asserting a counterclaim against the woman
for breach of contract. In addition, the company
timely served a demand for a jury trial. The
woman did not.
Which statement best describes the roles of
the judge and jury as finders of fact in the trial of
the parties’ claims?
(A) The judge will first determine the issues
relating to the woman’s claim for rescission,
and if it concludes the rescission is
not warranted, the jury will determine the
issues relating to the company’s breach-ofcontract
counterclaim.
(B) The jury will first determine the issues
relating to the breach of contract claim, and
the judge will determine the issues relating
to the rescission claim that have not already
been resolved by the jury.
(C) The jury will act as the sole finder of fact.
(D) The judge may either act as the sole finder
of fact on both the claim and the counterclaim,
or allow an advisory jury to try the
issues relating to the company’s counterclaim.

A

(B) If legal and equitable claims are joined in one action involving common fact issues, the legal claim is tried first before the jury, and then the equitable claim is tried to the court.

The jury’s finding on fact issues will bind the court in the equitable claim.

(A) is incorrect because all fact issues relating to the company’s legal claim must first be determined by the jury. Only then may the judge try the woman’s equitable claim.

(C) and (D) are both incorrect because neither the jury nor judge acts as the sole finder of fact when the case has legal claims and equitable claims, and a jury has been demanded on the legal claims.

How well did you know this?
1
Not at all
2
3
4
5
Perfectly
79
Q

A utility company constructed a building costing approximately $2 million that encroached on a rancher’s property. The rancher is suing the company in federal district court to force the company to remove the office building. In the same action, the rancher is asking for $200,000 in damages incurred because of the trespass. The court has diversity of citizenship jurisdiction. Is the company entitled to a jury trial?
(A) Yes, for all issues.
(B) Yes, but only for issues involving the
trespass action.
(C) Yes, but only for issues involving the action to require the company to move its building.
(D) No, because the equitable issue predominates.

A

(B) The company is entitled to a jury trial only for issues involving the trespass action.

In this case, both legal and equitable issues exist. The defendant is entitled to a jury trial on the legal issues in the case, even though the equitable issue of whether an injunction mandating removal of the building should be issued clearly predominates.

Hence, (D) is incorrect. (A) and (C) are incorrect
because, as stated, the issue regarding removal of the building is equitable, and thus there is no
right to answer it to a jury.

How well did you know this?
1
Not at all
2
3
4
5
Perfectly
80
Q

A restaurant owner properly sued a food
supplier in federal district court for breach of
contract and timely demanded a jury trial. The
complaint asserted both legal and equitable
claims.
Which of the following statements correctly
states the proper order for trying both claims?
(A) All legal claims should be tried first by the
jury.
(D) It is up to the federal district court judge’s
discretion which claim will be tried first.

A

(A) If legal and equitable claims are joined in one action involving common fact issues, the legal
claim should be tried first to the jury and then the equitable claim to the court (the jury’s finding
on fact issues will bind the court in the equitable claim).

(D) is wrong because the Supreme Court has held that if legal and equitable claims are joined in one action involving common fact issues, the legal claim should be tried first.

How well did you know this?
1
Not at all
2
3
4
5
Perfectly
81
Q

A gas station owner properly filed a breach of
a noncompete clause in an employment contract
case in federal court against a former manager,
who began building a similar gas station across
the street. The owner’s complaint requested
money damages and an injunction to stop the
construction. The former manager wants a jury trial, but the gas station owner prefers that the issue be litigated before the judge. Can the former manager demand a jury trial for both claims?
(A) Yes, because the Seventh Amendment gives
both the plaintiff and the defendant the
right to demand a jury trial in civil actions.
(D) No, because, while the Seventh Amendment
gives both the plaintiff and defendant
the right to demand a jury trial in civil
actions, when a case presents both legal and
equitable claims, the right is only preserved
for all legal claims, not equitable claims.

A

(D) The former manager may not demand a jury trial for both claims.

The Seventh Amendment preserves the right to a jury trial for both the plaintiff and defendant for civil actions in federal court of facts in all “suits of common law” where the amount in controversy exceeds $20.

The determination is historical and turns initially on whether the claim or relief was available at law or in equity in 1791. However, the Supreme Court has demonstrated a clear preference for jury trials in doubtful cases and has held that, if damages are claimed as part of an action seeking an injunction, the defendant cannot be denied a jury on the damages issues on the ground that they are “incidental” to the equitable relief. As such, when a case presents both legal and equitable claims, the right to a jury is preserved for all legal claims, but not for equitable claims. Instead, the legal claim should be tried first to the jury, and the equitable claim will go to the court (the jury’s finding on fact issues will bind the court in the equitable claims).

Here, the call of the question asks if the former manager can demand a jury trial for both claims. Under the Seventh Amendment, he only has a right to a jury trial on the legal claim for money damages, not the equitable claim for the injunction. Instead, the court will hear the equitable claim for the injunction, but will be bound by any of the jury’s factual decisions that relate to the equitable claim.

(A) is wrong because, although it is a correct statement of law, it is incomplete and does not directly answer the question asked. Both plaintiffs and defendants do have a right to demand a jury trial for civil actions in federal court, but in cases asserting both legal and equitable claims, the right to trial extends only to all issues that are relevant to the legal claims. Here, since the injunction is an equitable claim, the former manager does not have a right to a jury to review that claim.

How well did you know this?
1
Not at all
2
3
4
5
Perfectly
82
Q

A man left a tavern after consuming several
alcoholic drinks and then attempted to drive
home. On the way home, he was involved in
an accident with another driver, injuring him
severely.
The injured driver brought a negligence
action against the tavern owner and demanded
a jury trial. At trial, the driver called several
witnesses to testify that the tavern owner served
the man alcohol after it became apparent that
the man was intoxicated, but the tavern owner
refuted these assertions during his testimony. At
the appropriate time, both parties moved for a
judgment as a matter of law (“JMOL ”), and both
motions were denied. The jury then returned a
verdict for the driver.
If the tavern owner seeks to challenge the
jury verdict, what would be the proper and best
course of action?
(A) File a notice of appeal.
(B) File a motion for a new trial.
(C) File a renewed motion for a JMOL .
(D) File both a motion for a new trial and a
renewed motion for a JMOL .

A

(D) The proper course of action for the tavern owner would be to file a renewed motion for a JMOL and a motion for a new trial.

The trial judge will then rule on both motions, allowing the appellate court to review both motions on appeal. And note that a renewed motion for JMOL would be permitted since the tavern owner initially sought a JMOL at trial.

(C) is incorrect because a party should move for both a new trial and a motion for judgment
as a matter of law in order to preserve both for appeal.

How well did you know this?
1
Not at all
2
3
4
5
Perfectly
83
Q

For statute of limitations
purposes, an amendment to a pleading that arises from the same conduct, transaction, or
occurrence that was set forth (or was attempted to be set forth) in the original pleading generally
is deemed filed …

A

…. on the date that the original pleading was filed.

How well did you know this?
1
Not at all
2
3
4
5
Perfectly
84
Q

Under Rule 14, a defendant may assert a third-party claim against “a nonparty who is or may be liable to it for all or part of the claim against it.” In other words, a third-party claim must be …

A

… a derivative claim; the third-party plaintiff must be seeking indemnification or contribution from the third-party defendant.

i.e.

Where the gas worker’s claim is not that the electrician must indemnify him or that the electrician is a joint tortfeasor who may be jointly liable under principles of contribution; rather, the gas worker is alleging that he is not liable and that the electrician is; the claim is not derivative, and it is not properly asserted as a third-party claim under Rule 14.

How well did you know this?
1
Not at all
2
3
4
5
Perfectly
85
Q

A defendant may serve a third-party complaint
as of right within … days of serving his original answer.

A

14

How well did you know this?
1
Not at all
2
3
4
5
Perfectly
86
Q

A consumer purchased a luxury automobile
from a dealer on credit. After the consumer
failed to make a number of the required
payments, the dealer filed a civil action against
the consumer in federal district court to recover
the balance due on the account. The dealer
properly served process on the consumer.
Several months passed, and the consumer did not file any response to the complaint. The dealer
then filed a motion asking the clerk of court to
make an entry of default, and the clerk did so.
What procedure should the dealer follow to
obtain a default judgment against the consumer?

(A) File a motion to have the clerk of court enter
the default judgment, and the clerk may
do so without the consumer receiving any
further notice of the motion.
(C) File a motion to have the judge enter the
default judgment, and the judge may do so
without the consumer receiving any further
notice of the motion for default judgment.

A

(A) The dealer should file a motion with the clerk of court.

On request of the plaintiff, supported by an affidavit as to the amount due, the clerk may sign and enter judgment for that amount and
costs against the defendant if:

(i) the plaintiff’s claim against the defaulted defendant is for a sum certain;

(ii) the default was entered because the defendant failed to appear;

(iii) the defaulted defendant is not an infant or incompetent person; and

_(iv) the damages amount requested is not
greater than the amount requested in the complaint.
_

The dealer’s claim here meets these requirements, so the clerk may enter the default judgment, and no notice is required.

(C) is incorrect because the clerk may enter the default judgment when the amount is for a sum certain.

87
Q

A homeowner entered into a contract with a builder to construct an addition to the homeowner’s house. The builder engaged a subcontractor to perform the carpentry and framing work. The homeowner filed a breach of contract action against the builder for defective work on the house. The builder in turn impleaded the subcontractor, claiming the subcontractor was responsible for any defects and seeking indemnity for any sum to which the homeowner is entitled. The subcontractor some months before had done some carpentry work for the homeowner on an apartment building that the homeowner owned, and the subcontractor was never paid for that work. May the subcontractor file a breach of contract claim in the pending action to recover from the homeowner the money due for the apartment building work?

(B) No, because the claim does not arise from
the same transaction or occurrence as the
homeowner’s initial claim.
(D) Yes, because the subcontractor may at
his election assert his claim against the
homeowner in the pending action or he may
assert it as an independent action.

A

(B) The subcontractor cannot assert a breach of contract claim in the pending action.

A third-party defendant may assert a claim against the original plaintiff only if the claim arises from the same transaction or occurrence that is the subject of the plaintiff’s original claims.

Here, the contractor’s claim arises out of the work he did on the homeowner’s apartment building, but the homeowner’s claim arises out of the work the subcontractor did on the addition to the homeowner’s house. Therefore, the subcontractor cannot assert his claim in the pending action, and (D) is incorrect.

88
Q

A State A book dealer sold a number of rare
comic books to a State B collector for $90,000.
The collector paid $10,000 upon delivery and
agreed to pay the balance in eight monthly
installments of $10,000 each. Two weeks after
the sale, the dealer and the collector were
involved in an automobile accident that was
unrelated to the comic book transaction. The
dealer filed a negligence action against the
collector in federal district court to recover
$100,000 for injuries incurred in the automobile
accident.
If the collector wishes to assert a $90,000
fraud claim against the dealer on the grounds
that the comic books were counterfeit, may the
collector assert this claim in the pending negligence action?
(A) The collector must assert the fraud claim
against the dealer in a separate action because
the two claims do not arise from the
same transaction or occurrence.
(C) The collector may assert the fraud claim
either as a counterclaim or in a separate
lawsuit.

A

(C) The collector may assert the fraud claim either as a counterclaim or in a separate lawsuit.

The collector’s fraud claim is a permissive counterclaim since it does not arise from the same transaction or occurrence as the dealer’s claims. Therefore, the collector may assert it as a counterclaim or bring a separate lawsuit.

(A) is incorrect because a defendant may assert a permissive counterclaim, assuming there is subject matter jurisdiction, even though there is no connection between it and the plaintiff’s claim.

89
Q

A truck, a car, and a motorcycle were in a
three-vehicle accident. The operators of the
three vehicles are citizens of different states. The
motorcyclist filed a negligence action against
the truck driver in federal district court, seeking
$500,000 in damages. The truck driver asserted
a third-party claim against the car driver,
alleging that the car driver was also negligent
and should be liable in contribution for part of
any sum the truck driver had to pay the motorcyclist.
Can the motorcyclist now assert and maintain
in the pending action a negligence claim directly
against the car driver?
(A) No, because the motorcyclist did not assert
the claim against the car driver in his original
complaint.
(B) No, because the federal court lacks subject
matter jurisdiction over the motorcyclist’s
claim against the car driver.
(C) Yes, because the claim arises from the
same transaction or occurrence as the
motorcyclist’s original claim against the
truck driver.
(D) Yes, because the motorcyclist may assert
any claim he has against the car driver once
the car driver is made a party to the action.

A

(C) The motorcyclist can assert and maintain a negligence claim against the car driver in the pending action.

Under Rule 14(a)(3), a plaintiff may assert a claim against _a third-party defendant brought
in by a defendant’s third-party claim_only if the plaintiff’s claim against the third-party defendant arises from the same transaction or occurrence as the plaintiff’s original claim.

Because the motorcyclist’s claims against the truck driver and car driver both arise from the accident, he can assert a claim against the car driver.

(A) and (D) are therefore incorrect.

(B) is incorrect because the court has diversity of citizenship jurisdiction over the claim. The facts state that the three operators are all from different states, and, when looking at a joint claim asserted against multiple defendants, the court looks to the total value of the claim in determining the amount in controversy.
Here, the claim is for $500,000, well above the amount in controversy requirement for
diversity jurisdiction.

90
Q

In federal court, an amendment to a complaint “relates back” to the date the complaint was filed when …

A

… the new claim arises from the same transaction or occurrence as the original claim.

91
Q

A bus collided with a car. The bus company
hired an investigator to prepare a report
regarding the accident. The investigation
revealed that the bus driver was speeding
when the accident occurred. The investigator
included this information in his report to the bus
company.
The driver of the car sued the bus company
in federal court, properly invoking the court’s
diversity of citizenship jurisdiction. The driver
of the car served interrogatories on the bus
company, asking whether the bus driver was
speeding when the accident occurred.
Assuming that the investigator’s report is
the sole basis of the bus company’s knowledge
regarding the accident, must the company
disclose that the driver was speeding?
(A) No, because the report was prepared in
anticipation of litigation.

(D) Yes, because the information is relevant and
not subject to the work product exception.

A

(D) Work product protects documents and materials prepared in anticipation of litigation, not the underlying facts in the document itself.

Thus, regardless of whether the investigator’s report is work product, the bus company must nonetheless disclose all relevant facts regarding the accident, including the fact that the driver was speeding.

92
Q

When a party who notices a deposition fails to serve a subpoena on a nonparty deponent who then does not appear, the opposing party may recover …

A

… reasonable expenses for attending, including attorney’s fees.

93
Q

Interrogatories may be served only on …

A

… another party.

94
Q

A party may obtain, without a court order and without showing need and hardship, ….

A

…. a copy of any statement or recording previously made by that party.

95
Q

A buyer filed a breach of contract action against a seller in federal district court. The buyer alleges that the two parties had a contract under which the seller agreed to sell to the buyer a specified number of widgets at a specified price. The seller alleges that the parties had negotiated a possible sale but never reached an agreement. The seller thus denies the existence of a contract. The buyer spent several hundred dollars retrieving from its computer archives the email communication that took place between it and the seller. The buyer intends to use the emails as evidence that the parties did in fact have a contract. The retrieved emails currently are saved on a computer hard drive at the buyer’s office.
To what extent may the seller obtain discovery
regarding the emails?
(A) The buyer must provide the seller a copy
of the emails as part of the buyer’s initial
disclosures.
(B) Although the buyer is not required to
produce electronically stored information
in its required disclosures, the seller may
obtain a copy of the emails by serving an
appropriate request.

A

(A) The buyer must provide a copy of the emails as part of its initial disclosures.

Without waiting for a discovery request, a party must provide to the other parties copies or descriptions of electronically stored information that is in the disclosing party’s possession or control and that the disclosing party may use to support its claims or defenses. (B) is therefore incorrect.

* The work product doctrine protects materials made in anticipation of litigation, not materials retrieved in anticipation of litigation.

96
Q

A contractor filed a breach of contract action
against a supplier in federal district court,
seeking compensatory damages. The contractor
does not seek, and under applicable law cannot
recover, punitive damages. The supplier admits
the existence of the contract but denies breach.
The contractor served on the supplier an interrogatory asking the supplier to state his net
worth. Assuming the supplier objects to the interrogatory and the contractor files a motion to compel an answer, will the court require the supplier to answer?
(A) Yes, because the requested information is
relevant.
(D) No, because the requested information is
not relevant to the claim or defense of any
party.

A

(D) The court will not require the supplier to answer.

In general, discovery may be had of any
nonprivileged matter that is relevant to any party’s claim or defense and proportional to the needs of the case.

Here, the supplier’s net worth is not relevant to any party’s claim or defense because the contractor does not seek punitive damages. Thus, (A) is incorrect.

97
Q

A party may serve on a nonparty a …. that
compels the nonparty to produce physical material, including documents and electronically stored
information, relevant to the pending action.

A

subpoena

98
Q

The defendant was involved in a threecar
accident with two other drivers. The first
driver’s car was totaled and had an estimated
replacement cost of more than $75,000. The
second driver’s car was also totaled, and it had
an estimated replacement cost of $30,000, and
her medical bills were $50,000. The first driver
sued the defendant in federal court and served
a request for admission on him, asking that he
admit he was exceeding the posted speed limit
by 50 miles per hour. The defendant admitted
that fact. Thereafter, the second driver also
sued the defendant in federal court in a separate
action.
May the second driver use the admission by
the defendant in her case?
(A) Yes, because the admission is not considered
to be hearsay.
(B) Yes, because the mutuality requirement
for issue preclusion has largely been eliminated.
(C) No, because the issue was never actually
litigated.
(D) No, because admissions may be used in the
current litigation only.

A

(D) Under Rule 36 of the Federal Rules of Civil Procedure, an admission may not be used against the admitting party in any other proceeding.

(A) is incorrect. Although admissions are not considered to be hearsay, the hearsay rule would not govern the admissibility in this instance. (In other words, the admissions contained in a response to a request of admissions would not be admissible in another proceeding regardless of whether or not they are considered to be hearsay.)

(B) is a factor to be considered for the application of collateral estoppel and is not relevant to the current question.

Likewise, (C) is incorrect. Although it is true that an issue must be “actually litigated” for issue preclusion (collateral estoppel) to be available, it is arguable that submitting the admission
constitutes litigation. However, Rule 36 specifically bars use of an admission in another
proceeding, making (D) a better answer.

99
Q

The plaintiff filed a negligence action against the defendant to recover compensatory damages for injuries sustained in a car accident. The defendant believes that the plaintiff’s key witness has such poor eyesight that she could not
possibly have seen the things that she claims to
have seen. May the defendant require the witness to undergo an eye exam as part of discovery?

(C) No, because the witness is not a party to the
action.
(D) No, because the discovery rules do not
provide for obtaining any sort of physical
exam against the wishes of the person
examined.

A

(C) The defendant may not require the witness to undergo an eye exam because the witness is not a party to the action.

A party may request a physical or mental exam of another party when that party’s physical or mental condition is in controversy. The exam must be ordered by the court on a showing of good cause.

(As a side note, remember that a nonparty may be deposed. During the deposition, the witness may be asked about her medical condition. Furthermore, it also might be possible to obtain copies of the witness’s medical records by means of a subpoena or subpoena duces tecum.)

(D) is incorrect because the court may order a party to undergo a physical exam against his wishes if there has been a showing of good cause.

100
Q

The opinions of experts who are retained in anticipation of litigation but who are not expected to testify at trial may be discovered only …

A

… upon a showing of exceptional circumstances under which it is impracticable to obtain facts or opinions by other means or when a medical report for an exam conducted under FRCP 35 is requested.

101
Q

A mechanic sued his former employer in federal court, claiming that the employer had discharged him because of his age in violation of federal law. The employer answered, denying the claims and promptly moving for summary judgment. In support of the motion, the employer attached the mechanic’s employment evaluations for the past three years, which rated his skills and performance as poor and culminated in a recommendation for his discharge.

What is the mechanic’s best argument to defeat the summary judgment motion?

A. The allegations in the complaint conflict with the mechanic’s employment evaluations, raising a genuine dispute as to material facts.

B. The employer cannot rely in his motion on matters outside the pleadings.

C. The essential facts are unavailable to the mechanic and therefore discovery is required.

D. The motion was filed before the close of discovery.

A

(C) is correct.

If the mechanic (the nonmovant) shows by affidavit or declaration that he cannot present facts essential to justify his opposition to the summary judgment motion, Rule 56(d) authorizes him to ask the court to defer action or deny the motion to allow time to obtain affidavits or declarations or to take discovery.

The employer moved for summary judgment right after answering and before any discovery. That timing would support defeating the summary judgment motion at this time.

(A) is incorrect.

Under Rule 56(c), a party asserting that a fact is genuinely disputed must support the assertion by citing particular parts of the record, including affidavits or declarations, stipulations, or discovery materials. The mechanic cannot simply rely on the complaint allegations to rebut the employer’s evidence but must support his factual position with his own evidence that a factual dispute exists. If he cannot do so, Rule 56(d) authorizes him to ask the court to defer action or deny the summary judgment motion to allow time to obtain affidavits or declarations or to take discovery.

(B) is incorrect. The function of a summary judgment motion is to allow additional evidence outside the pleadings to show that there is no genuine dispute of fact and that the movant is entitled to judgment as a matter of law. Rule 56(c)(1) enumerates the types of materials that the moving party may use to support a summary judgment motion, including documents. If the mechanic (the nonmovant) shows by affidavit or declaration that he cannot present facts essential to justify his opposition to the summary judgment motion, Rule 56(d) authorizes him to ask the court to defer action or deny the motion to allow time to obtain affidavits or declarations or to take discovery.

(D) is incorrect. The fact that a summary judgment motion is filed before the close of discovery does not require the court to deny it.

Under Rule 56(b), a party may file the motion at any time until 30 days after the close of all discovery. The problem here is that the employer filed the motion before discovery commenced, thus providing the mechanic an argument to defer action or deny the motion under Rule 56(d).

102
Q

A shop owner domiciled in State A sued a distributor in a federal district court in State A for breach of a contract. The shop owner sought $100,000 in damages for allegedly defective goods that the distributor had provided under the contract. The distributor is incorporated in State B, with its principal place of business in State C. The distributor brought in as a third-party defendant the wholesaler that had provided the goods to the distributor, alleging that the wholesaler had a duty to indemnify the distributor for any damages recovered by the shop owner. The wholesaler is incorporated in State B, with its principal place of business in State A.

The wholesaler has asserted a $60,000 counterclaim against the distributor for payment for the goods at issue, and the distributor has moved to dismiss the counterclaim for lack of subject-matter jurisdiction.

Should the motion to dismiss be granted?

A. No, because the wholesaler’s and the distributor’s principal places of business are diverse.

B. No, because there is supplemental jurisdiction over the wholesaler’s counterclaim.

C. Yes, because there is no diversity of citizenship between the distributor and the wholesaler.

D. Yes, because there is no diversity of citizenship between the shop owner and the wholesaler.

A

(B) is correct.

There is supplemental jurisdiction because the claim for nonpayment for the goods in issue arises out of a common nucleus of operative fact as the plaintiff’s (the shop owner’s) claims, and the restrictions contained in the supplemental jurisdiction statute do not apply to these facts.

This question tests on the restrictions placed on the use of supplemental jurisdiction when the case is in federal court under diversity jurisdiction and the use of supplemental jurisdiction is inconsistent with the requirements for diversity.

The rule to remember here is this:

The restrictions on the use of supplemental jurisdiction apply to plaintiffs only.

Here, the defendant (the distributor) is the party using supplemental jurisdiction, so the restrictions do not apply. As a result, it does not matter that the defendant and third-party defendant are not diverse, and it does not matter that the amount in controversy requirement is not satisfied.

(A) is an incorrect statement of the law. A corporation is a citizen of each state in which it is incorporated (only one in real life) and the one state in which it has its principal place of business. Since the wholesaler and the distributor are both incorporated in State B, complete diversity of citizenship does not exist.

(C) is incorrect. Although it is a true statement of law, the answer ignores the potential for supplemental jurisdiction.

(D) is incorrect. The shop owner is not asserting any claims against the wholesaler, so the lack of diversity between them is irrelevant.

103
Q

An individual investor purchased stock through a company’s stock offering. When the price of the stock plummeted, the investor sued the company in a state court in State A, claiming that the company’s offering materials had fraudulently induced him to purchase the stock and seeking $25,000 in damages.

A university that had purchased the company’s stock through the same offering sued the company in federal court in State B, claiming that the offering materials violated federal securities laws and seeking $1 million in damages.

The individual investor’s suit proceeded to trial. The state court ruled that the company’s offering materials contained false information and awarded the investor a $25,000 judgment. The university immediately moved for partial summary judgment in its federal action against the company, arguing that the state court judgment bound the federal court on the issue of whether the company’s offering materials contained false information.

Neither State A nor State B permits nonmutual issue preclusion.

Should the court grant the university’s motion?

A. No, because State A does not permit nonmutual issue preclusion.

B. No, because the federal court sits in a state that does not permit nonmutual issue preclusion.

C. Yes, because federal law permits nonmutual issue preclusion.

D. Yes, because the issue of whether the materials contained false information was actually litigated and necessarily decided.

A

(A) is correct.

Generally speaking, when dealing with the preclusive effect of a judgment, the recognizing court should not give a judgment any greater effect than the rendering state would. (When “case one” has been decided in state court, the court in case two generally will apply the claim or issue preclusion of the jurisdiction that decided case one.)

Here, the rendering state, State A, does not recognize nonmutual collateral estoppel, so a federal court sitting in State B should not give the judgment issue preclusive effect.

(B) is incorrect. As stated above, the law of the rendering state generally determines the preclusive effect of the judgment.

(C) is incorrect for a similar reason. The fact that federal law may permit nonmutual issue preclusion is irrelevant. The law of the rendering state generally controls the preclusive effect of the judgment.

(D) is incorrect. The listed factors are used to determine whether a court may apply issue preclusion, but they do not address whether nonmutual issue preclusion is applicable.

***

Issue precclusion is “non-mutual” when it is not between former parties or their privies. Although non-mutual collateral estoppel (“NMCE”) is sometimes permitted, Due Process prevents its use against a person who themselves (or their privy) never litigated the issue in the prior suit.

104
Q

A man filed a federal diversity action against a bus company, seeking damages for injuries he had sustained in an accident while riding a bus owned by the company. The man demanded a jury trial.

After the parties’ attorneys examined the prospective jurors and exercised their challenges, six jurors and two alternate jurors were chosen. During the trial, two jurors became ill and were replaced by the alternate jurors. At the conclusion of the trial, a third juror also became ill, and the court excused that juror.

The parties’ attorneys stipulated to the return of a verdict from a five-person jury. The jury then deliberated and returned a verdict for the company. The man timely filed a motion for a new trial, arguing that the five-person jury was not large enough to return a verdict.

Should the court grant the motion?

A. No, because the court properly excused the three jurors due to illness.

B. No, because the parties stipulated to a verdict from a jury of fewer than six jurors.

C. Yes, because there must be at least six jurors on a federal civil jury.

D. Yes, because there must be at least 12 jurors on a federal civil jury.

A

(B) is correct. A jury must be composed of at least six jurors at the beginning of the trial. If the number drops below six (for example, as here, when jurors became ill), a mistrial results, unless the parties agree to a lesser number of jurors. Because the parties stipulated to fewer jurors, the issue is waived.

(A) is incorrect because even though the jurors were properly excused, a lesser number of jurors cannot return a verdict unless the parties agree.

(C) is incorrect because the parties may stipulate to fewer jurors.

(D) is incorrect. A jury must be composed of at least six jurors, not 12, and the answer does not address the possibility of a stipulation.

105
Q

A small commercial airplane crashed in State A. The passengers and pilot, all citizens of State B, were killed in the crash. The airline that owned and operated the airplane is incorporated and has its maintenance facilities and principal place of business in State C.

One day before the statute of limitations on their claims would have run, the estates of the pilot and each of the passengers filed a wrongful death action against the airline in federal court in State A. The airline was served one week later and wants to prevent the State A federal court from hearing the action.

Which of the following motions is most likely to accomplish the airline’s goal?

A. A motion to dismiss the action for improper venue.

B. A motion to dismiss the action for lack of personal jurisdiction.

C. A motion to dismiss the action under the doctrine of forum non conveniens.

D. A motion to transfer the action to a federal court in State C.

A

(D) is correct, and (A), (B), and (C) are incorrect for essentially similar reasoning. The best approach to answer this question is to use a process of elimination.

In order to hear a case, a federal court must have subject matter jurisdiction and personal jurisdiction over the defendant, and venue must be proper. However, even if venue is proper, the court may transfer the case to another federal court, in the interests of justice, for the convenience of the parties and witnesses. Venue is proper in the judicial district in which a defendant resides, if all defendants reside in the same state, and in any judicial district where a substantial part of the acts or omissions took place or where a substantial part of the property that is the subject of the litigation is located.

Here, the airplane crashed in State A, so a substantial part of the acts or omissions took place there. Thus, venue is proper, and (A) is incorrect.

(B) is likewise incorrect because the airplane crashed in State A. Since the airplane crashed in State A, the airline likely committed a tort in State A, thus giving a State A court personal jurisdiction over the defendant airline under the state’s long arm statute. Thus, a motion to dismiss on the basis of a lack of personal jurisdiction is unlikely to be successful.

Finally, (C) is incorrect because the fact that the plane crashed in State A would likely defeat a motion to dismiss based on forum non conveniens. Federal courts typically will transfer (rather than dismiss) a case for convenience reasons under 28 U.S.C. section 1404 when the more convenient forum lies in another federal judicial district. A dismissal based on forum non conveniens usually is used when the federal court cannot transfer the action, such as when the more convenient forum is in a foreign country.

Here, since all proper venues are within the United States, there is no reason the court could not transfer the case. By process of elimination,

(D) is correct. The court could transfer to a federal court in State C based on convenience factors. It might be a tough sell for the airline to have the plaintiff’s choice of forum overridden based on its domicile only, but it is the only possible way to have the case heard in a federal court other than in State A.

106
Q

Under Federal Rule of Civil Procedure 51(d)(2), a court may consider a plain error in the jury instructions that has not been preserved by an objection if …

A

… the error affects a substantial right.

107
Q

Before responding to a pleading or, if no responsive pleading is permitted, within … days after service of the pleading, a party may move to have stricken any insufficient defense, or any redundant, immaterial, impertinent, or scandalous matter.

A

21

108
Q

A patient domiciled in State A sued a surgeon domiciled in State B in a federal court in State A, alleging claims for malpractice. The surgeon moved to dismiss the action for lack of personal jurisdiction. The court denied the motion and set discovery cutoff and trial dates.

The surgeon has appealed the denial of the motion.

Should the appellate court hear the merits of the surgeon’s appeal?

A. No, because the appellate court lacks jurisdiction over the appeal.

B. No, because the district court’s decision on jurisdiction is final.

C. Yes, because a contrary appellate decision could terminate the action.

D. Yes, because the surgeon’s personal-jurisdiction challenge raises a constitutional question.

A

(A) is correct.

With certain exceptions by rule or statute, only final judgments may be appealed. A final judgment is one that disposes of the whole case on its merits. Since the case was still *pending* after the denial of a motion to dismiss on the basis of a lack of personal jurisdiction, the judgment here was not a final judgment and thus was not appealable.

(B) is incorrect because an order determining personal jurisdiction may be appealed (and is not within the sole discretion of the district court), but the order must be “final” (that is, it must dispose of all claims by all parties). (C) is an incorrect statement of law and the final order rule. The order must be “final” at the trial court level, not be potentially made final by an appellate court ruling. (D) again is an incorrect statement of law. The final order rule may not be circumvented by the mere raising of a constitutional issue.

109
Q

A football team entered into a 10-year lease with a city for use of the city’s athletic stadium. Five years into the lease, the team threatened to leave the stadium and move to another city.

The city sued the team in federal court, seeking a permanent injunction to prevent the team from breaching its lease and leaving. In its answer, the team included a counterclaim seeking $10 million in damages for losses caused by the city’s alleged failure to properly maintain the stadium, as the lease required. The team demanded a jury trial on the counterclaim.

The city moved to try its claim for a permanent injunction before the trial on the team’s counterclaim. The team objected and moved that the jury trial of its counterclaim be held before the trial of the city’s injunction claim.

How should the court rule on the parties’ motions?

A. The court should first hold a jury trial of the team’s counterclaim, and then a nonjury trial of the issues remaining in the city’s claim.

B. The court should first hold a nonjury trial of the city’s claim without giving binding effect to its findings or conclusions in the later jury trial of the team’s counterclaim.

C. The court should first hold a nonjury trial of the city’s claim, and then a jury trial of the issues remaining in the team’s counterclaim.

D. The court should schedule a jury trial of both the city’s claim and the team’s counterclaim.

A

(A) is correct.

If legal and equitable claims are joined in one action involving common fact issues, the legal claim should be tried first to the jury and then the equitable claim to the court, but the jury’s finding on fact issues will bind the court in the equitable claim.

(B) is incorrect because only one trial is held. As stated, the jury’s findings on fact issues will bind the court in the equitable claim.

(C) is incorrect because a party cannot be denied a jury trial on “legal” issues even though there may be mixed “legal” and “equitable” claims in the case.

(D) is incorrect because a party is not entitled to a jury on equitable claims.

110
Q

A court may grant, in its discretion, an ex parte temporary restraining order if the moving party

A

(i) gives specific facts in an affidavit or in the verified complaint to establish that immediate and irreparable injury will result to the moving party before the adverse party can be heard in opposition
(ii) certify in writing all efforts she made to give notice of the hearing to the adverse party and the reasons why notice should not be required; and
(iii) provide some security, the amount of which is determined by the court, to pay for any costs and damages incurred by the adverse party if he was wrongfully enjoined or restrained.

The United States, its officers, and its agencies are not required to give security.

111
Q

The Seventh Amendment provides the right to a jury trial in …

A

… federal courts for the determination of facts in all suits at common law where the amount in controversy exceeds $20.

The Supreme Court has held that when legal and equitable claims are joined in one action, the legal claim should be tried first to the jury and then the equitable claim should be decided by the court. The jury’s fact determinations bind the judge

112
Q

A motion for relief from judgment because the judgment is void must be brought within …

When relief is based on

(i) mistake, inadvertence, surprise, or excusable neglect;
(ii) newly discovered evidence that, by due diligence, could not have been discovered in time to move for a new trial; or
(iii) fraud, misrepresentation, or other misconduct of an adverse party, the motion must be brought within ….

A

… a reasonable time, but there is no outermost deadline.

…. a reasonable time not to exceed one year.

113
Q

The Supreme Court may hear:

A

The Supreme Court may hear some appeals from state courts and some appeals directly from federal district courts.

The Supreme Court hears, on direct appeal, any order granting or denying an injunction in any proceeding required to be heard by a three-judge district panel.

It also may hear by discretionary writ of certiorari final judgments of the highest court of a state if:

(i) the validity of a treaty or federal statute is drawn into question;
(ii) the validity of a state statute is drawn into question on the ground that it is repugnant to the federal Constitution or to a treaty or federal statute; or
(iii) any title, right, privilege, or immunity is claimed under the federal Constitution or treaty or federal statute.

114
Q

Under the Full Faith and Credit Clause of the Constitution and federal statutes ….

A

… a state court generally must recognize the judgments of a sister state and federal courts generally must recognize judgments of state courts. Recognition of judgments is required between state courts, between state and federal courts, and between federal courts.

115
Q

Supplelemtal jurisdiction

A

Once one claim satisfies the requirements for original federal subject matter jurisdiction, the court has discretion to exercise supplemental jurisdiction over related claims that derive from the same common nucleus of fact and are such that a plaintiff would ordinarily be expected to try them in a single judicial proceeding

116
Q

If the plaintiff bases the motion to remand on a defect other than subject matter jurisdiction (i.e., a defect in the removal procedure), the motion to remand must be brought within … days of removal. There is no such time limit for a lack of subject matter jurisdiction. The court must remand whenever it is shown that there is no subject matter jurisdiction.

A

30

117
Q

Under issue preclusion (collateral estoppel), …

A

… a judgment binds the plaintiff or defendant (or their privies) in subsequent actions on different causes of action between them (or their privies) as to issues actually litigated and essential to the judgment in the first action. Issue preclusion focuses on something relatively narrow–an issue that was litigated and determined in the first case, and that is relevant in a second case. With issue preclusion, the issue is deemed established in the second case without need to proffer evidence on it.

118
Q

Under the traditional “mutuality” rule, only someone who was a party (or in privity with a party) in the previous case can use issue preclusion.

A

However, this requirement is not imposed by due process and has been subject to modification in certain circumstances to allow nonparties to take advantage of a prior judgment. In fact, although courts have been very reluctant to permit a nonparty to use issue preclusion to aid him offensively (as a plaintiff) to obtain relief, the Supreme Court has upheld its use offensively by a nonparty where it was fair and equitable to do so. [Parklane Hosiery Co. v. Shore, 439 U.S. 322 (1979)]

119
Q

On motion and just terms, the court may relieve a party from a final judgment or order on the following grounds:

A

(i) mistake, inadvertence, surprise, or excusable neglect;

(ii) newly discovered evidence that by due diligence could not have been discovered in time to move for a new trial;

(iii) fraud, misrepresentation, or other misconduct of an adverse party;

(iv) the judgment is void;
(v) the judgment has been satisfied, released, or discharged;

a prior judgment on which it is based has been reversed or otherwise vacated;

or it is no longer equitable that the judgment should have prospective application; or

(vi) any other reason justifying relief from the operation of the judgment.

For grounds (i), (ii), and (iii), the motion must be made within a reasonable time not to exceed one year from the judgment; for the other grounds, the motion must be made within a reasonable time.

(But remember that a lack of subject matter jurisdiction may be raised at any time.) Such a motion is left to the trial judge’s discretion, and, on appeal, her decision will be reviewed on an “abuse of discretion” standard.

120
Q

Stay of Proceedings to Enforce a Judgment

A

Absent a court order, no execution on a judgment is allowed for 30 days after entry e_xcept for injunctions and receiverships, which are not held up_ unless otherwise ordered by a court.

121
Q

The court may continue to exercise supplemental (pendent) jurisdiction over the state claim even though ….

A

….the federal claim is dismissed on the merits.

However, the state claim should probably also be dismissed (without prejudice) if the federal claim is dismissed before trial.

i.e.

Here, although the federal copyright claim was invalid, it was deemed to be so during trial, before a verdict was rendered. Since the case is currently being tried, the court will likely hear the state claim for the sake of judicial economy.

122
Q

The federal courts and the state courts have concurrent jurisdiction over most types of actions. The few areas over which the federal courts have exclusive jurisdiction include:

A

bankruptcy cases,

patent and copyright cases,

antitrust cases, and a few other less common types of actions.

123
Q

When does federal question juridiction not exist?

A

The court does not have federal question jurisdiction when the recently enacted federal statute arises only in anticipation of the defendant’s defense.

(Rationale: If, for example, the defendant relied on some other defense or defaulted instead of defending on the basis of the new federal statute, no federal question would ever be involved in the case. [See Louisville & Nashville Railroad v. Mottley, 211 U.S. 149 (1908)])

124
Q

The opinion of an expert consulted in anticipation of litigation is discoverable if a party intends to call the expert as a trial witness. Conversely, the opinion of an expert who is retained in anticipation of litigation but who is not expected to testify at trial (i.e., a consulting expert)…

A

…is discoverable only upon a showing of exceptional circumstances under which it is impracticable to obtain facts or opinions by other means.

Absent such circumstances, these opinions are not discoverable, regardless of whether the nontestifying expert was informally consulted or whether she was retained or specially employed.

125
Q

Generally, a party may obtain discovery regarding …

A

…any matter that is proportional to the needs of the case and that is relevant to any party’s claim or defense, provided the matter is not subject to a privilege or to the exception for trial preparation materials, also called work product. Trial preparation materials are documents or other tangible things prepared in anticipation of litigation or for trial.

i.e.

Here, the manager’s report is relevant because it contains facts relating to the incident that is the basis of plaintiff’s claim. Moreover, it is not subject to the exception for trial preparation materials because it was prepared in the ordinary course of business to comply with state law, not in anticipation for litigation. Finally, the costs of producing the report likely would not be prohibitive.

126
Q

Federal trial courts are required to apply … when considering a motion for a new trial based on the excessiveness of the verdict.

A

a state law standard

127
Q

Under Rule 38, to receive a jury trial a party must …

A

… file a written demand and serve it on all the parties within 14 days after the service of the last pleading directed to the jury-triable issue.

Otherwise, the right to a jury trial generally is deemed waived.

Note, however, that the Supreme Court has held that, in the absence of compelling reasons to the contrary, a court should grant relief from the waiver if the issue is normally tried by a jury.

128
Q

Motion to dismiss

A

The defendant may file a motion and raise any or all of the following defenses:

(i) lack of subject matter jurisdiction;

(ii) lack of personal jurisdiction;

(iii) improper venue;

(iv) insufficiency of process;

(v) insufficiency of service of process;

(vi) failure to state a claim upon which relief can be granted; or

(vii) failure to join an indispensable party.

The defendant must raise defenses (ii) through (v) the first time he files a motion or
answer (or amendment as of right thereto)—whichever is first. If he does not, the defendant
waives these defenses.

Defenses (vi) and (vii) can be made at any time prior to trial or at trial.

Defense (i) can be raised at any time until all appeals have been exhausted.

A motion to dismiss for failure to state a cause of action that raises issues outside of the
pleadings will be treated as a motion for summary judgment.

129
Q

When an appellate court reviews a trial court’s ruling on a post-trial motion for judgment as a matter of law (including a renewed one), it employs … standard

A

de novo

It does so because the issue is one of law.

In contrast, when an appellate court reviews a trial court’s denial of a motion for new trial, it employs a more deferential standard, reversing the trial court’s denial only when there is a clear showing of an abuse of discretion.

Note that when a renewed motion for judgment as a matter of law and a motion for a new trial are made in the alternative and the renewed motion is granted, the court rules hypothetically on the new trial motion so that no remand is required if the ruling on the judgment as a matter of law is subsequently reversed on appeal.

130
Q

In a properly removed case, venue is proper in

A

the federal court of the state where the case was pending, even if venue would have been improper had the plaintiff originally filed the action in the federal district court of that state. This is because venue for an action removed under section 1441(a) lies in the federal district court “embracing the place where such [state] action is pending.”

131
Q

There are essentially two time restrictions on removal of a diversity case to federal court:

A

(1) a case based on diversity must be removed within 30 days of the defendant’s receipt of a copy of the paper (order, motion, etc.) that makes the case removable;

but (2) in no event may the case be removed more than one year after it was commenced in state court. [28 U.S.C. §1446]

132
Q

While driving in a city in State A, a citizen of State B struck a pedestrian who was a citizen of State C. The pedestrian sued both the driver and the State A city in federal district court, seeking $100,000. The pedestrian alleged that the driver was liable for negligently operating his car and that the State A city was liable for negligently maintaining a traffic signal. The driver, who owns an office supply wholesaler, also has a contract claim against the State A city for $80,000 worth of furniture that the city allegedly purchased and received but never paid for. Assume that State A has waived any applicable governmental immunity.

If the driver files a cross-claim for negligence against the State A city to recover damages for his injuries in the accident with the pedestrian, can the driver join with that cross-claim his contract claim for the purchase price of the furniture and maintain the claim in the same federal action?

A

The driver can join his contract claim with his negligence cross-claim, but he is not required to do so. As a general rule, a party may assert a cross-claim against a co-party only if the cross-claim arises from the same transaction or occurrence as that of the original action or of a counterclaim. However, once the party has filed such a cross-claim, he also may join with it any other claim that he has against the same party. Here, the driver’s contract claim is unrelated to the pedestrian’s negligence claim; however, the driver’s negligence claim is related to the pedestrian’s negligence claim. Thus, the contract claim also can be asserted in this case.

133
Q

A manufacturer of widgets was incorporated in and has its principal place of business in State A. The manufacturer also operates its own stores in State A, State B, and State C, through which it sells its widgets to consumers. A consumer who resides in State D visited a State D store to purchase a widget. The State D store did not have the widget he needed, and employees at the store directed him to the State B store operated by the manufacturer. The consumer visited the manufacturer’s State B store and purchased the widget. The consumer returned to his home in State D, where he was injured while using the widget. The consumer intends to file a products liability action against the manufacturer in federal district court.

In what district or districts is venue proper?

A

Venue is proper in the District of State A, the District of State D, and the District of State B.

Federal venue in civil actions is proper in

(i) the district where any defendant resides, if all defendants are residents of the state in which the district is located; and
(ii) the district in which a substantial part of the events or omissions giving rise to the claim occurred.

The District of State D is proper because a substantial part of the events giving rise to the claim—use of the widget and the injury—took place there.

Substantial events also occurred in the District of State B (the sale).

Additionally, venue is proper in other districts because the manufacturer is deemed to reside there. Corporations are deemed to reside in any district in which it is subject to personal jurisdiction with respect to the action in question. The manufacturer is subject to personal jurisdiction and thus deemed to reside in the District of State A (because its incorporation and principal place of business are there, thus subjecting it to general jurisdiction there), the District of State B (because it sold the widget from its store there, thus subjecting it to specific jurisdiction there with respect to the current action), and the District of State D (because the injury took place in State D, thus subjecting it to specific jurisdiction there with respect to the current action).

134
Q

Using his cellphone, a witness recorded a speeding driver hitting a pedestrian. The witness sold the recording to the driver. The driver then gave the recording to his attorney. After the pedestrian filed suit against the driver, the pedestrian sent a discovery request to the driver requesting that he produce “all items that show or describe the accident.”

Which of the following best describes whether the driver must provide the video, or a copy of it?

A

The driver must provide the video. Parties are entitled to discovery that fits under Rule 26(b) (1), which includes “any nonprivileged matter that is relevant to any party’s claim or defense.” In other words, even if the matter is only relevant to the opposing party, it would still be covered. Additionally, Rule 34 requires a party to produce relevant physical material, including electronically stored information, such as the recording here. There is no exception to relevance for matter that is damaging to a party, so (B) is incorrect. (Being damaging to the defendant’s defense might be reason not to disclose it as an initial disclosure because the defendant would not use the recording to support his defense, but the recording would have to be disclosed on a proper request.)

135
Q

An automotive engineer announced that he had developed a carburetor that will enable cars to achieve 100 miles per gallon of fuel, and that he will allow the carburetor to be inspected next month. Soon after, a former employer of the engineer brought an action to prohibit the engineer from displaying the carburetor, claiming that the engineer probably had stolen the carburetor’s design from the employer. The court granted the employer a temporary restraining order prohibiting the engineer from disclosing any mechanical details of his carburetor, and ordered a hearing to be held in one week to determine whether a preliminary injunction should be issued. Because each party would have to reveal the mechanical details of his designs at the hearing, the employer requested that the hearing be closed to the public and that the record be sealed to avoid revelation of his designs. The court granted the request. A reporter for a monthly automobile magazine heard about the case and wanted to attend the hearing. When he was told that the hearing would be closed, he filed an action to have it opened.

What is the reporter’s best argument for opening the hearing?

A Closure is not necessary to preserve an overriding interest here.

B The right of freedom of the press is extensive and allows the press to attend all hearings of interest to the public.

C Closure here amounts to a prior restraint.

D Under the fairness doctrine, the magazine will be required to give each litigant an opportunity to present his side of the case.

A

The reporter’s best argument is that the closure here is not necessary to preserve an overriding interest because trials and pretrial hearings generally must be open to the public.

The Supreme Court has held, at least in the context of criminal cases, that trials and pretrial proceedings can be closed only if closure is necessary to preserve an overriding interest and the closure order is narrowly tailored to serve the overriding interest.

While the Court has not yet established the standard for civil matters such as the case here, several Justices and commentators have suggested that the same standard will be applied in civil cases since they too have historically been open to the public.

136
Q

A credit card company filed a civil action against a consumer in federal district court seeking to recover the unpaid balance on the consumer’s account. The credit card company properly served process on the consumer, but the consumer failed to file or serve a timely answer to the complaint. The clerk of court on motion of the credit card company made an entry of default, but default judgment has not been entered.

On which grounds may the court set aside the entry of default?

C If the consumer demonstrates that there was good cause for his failure to file and serve a timely answer and that he has a viable defense.

D On any grounds that the court, in its discretion, finds just.

A

An entry of default may be set aside for “good cause shown.” Although not specifically required by the Federal Rules, a majority of courts also will require some showing of a meritorious defense.

(D) is incorrect because a majority of courts will also require a showing of a meritorious defense.

137
Q

A landlord brought suit against a tenant in federal court for overdue rent payments on a commercial lease. The landlord sought to recover on the six rent installments that were past due and unpaid at the time of the suit. The landlord won the case, and judgment was entered in her favor.

The lease has an acceleration clause that states that all future rent payments become due if the tenant falls behind three months or more. The landlord now files suit against the tenant for the remaining rent payments. The tenant moves to dismiss, asserting that the landlord’s claim is barred by claim preclusion (res judicata) principles.

Should the tenant’s motion to dismiss be granted?

A No, because a landlord may choose when to sue on an acceleration clause.

B No, because the two suits do not involve the same cause of action.

C Yes, because a landlord may not sue on later installments of an installment contract.

D Yes, because the two suits arose out of the same transaction or occurrence.

A

The plaintiff’s suit is unlikely to be successful because the two suits arise out of the same transaction or occurrence.

Claim preclusion (res judicata) requires that

(i) a valid, final judgment on the merits was entered in the first case;

(ii) the cases were brought by the same claimant against the same defendant; and

(iii) the same cause of action is involved in the later lawsuit.

Generally, a claimant is required to assert all causes of action arising out of the same transaction or occurrence that is the subject matter of the claim. I

n the situation of installment payments, the claimant is required to sue on all installments due at the time of the suit. If there is an acceleration clause, the claimant must sue for all installments.

Here, the plaintiff should have sued for all installments in the first lawsuit. Because she did not, her second suit is barred by claim preclusion.

138
Q

Under Rule 24, a nonparty may intervene in an action as a matter of right in two situations.

A

First, a nonparty may intervene when it has an unconditional right to do so by a federal statute.

Second, a nonparty may intervene if

(i) it has an interest in the property or transaction the is the subject matter of the action;
(ii) the disposition or resolution of the action may as a practical matter impair the nonparty’s ability to protect its interest; and
(iii) the nonparty’s interest is not adequately protected by an existing party in the action.

139
Q

A State A citizen filed a civil action against a State B citizen in a State B state court. The action arose from events that took place in State C. State C has only one federal district court (the District of State C). State B has two, the Northern District of State B and the Southern District of State B. The State B citizen resides in the Southern District of State B, while the state court action filed by the State A citizen is pending in a court located in the Northern District of State B.

If the State B citizen wishes to remove the action to federal district court, in which federal district should the State B citizen file a notice of removal?

A Either the Southern District of State B, the Northern District of State B, or the District of State C.

B Either the Southern District of State B or the District of State C.

C Either the Southern District of State B or the Northern District of State B.

D The Northern District of State B only.

A

The notice of removal should be filed in the Northern District of State B only.

The federal removal statute provides that the notice of removal should be filed in the federal district court for the district that geographically encompasses the state court from which the action is being removed. Thus, (D) is correct. (A) incorrectly states that a notice of removal may be filed in the federal district in which a substantial part of a lawsuit took place or in any federal district of the state in which the defendant resides. (B) incorrectly states that a notice of removal may be filed in the federal district in which the events took place or in which the defendant resides. (C) incorrectly states that the notice of removal may be filed in any district of the state in which the defendant resides

140
Q

An elderly woman filed a complaint in federal district court, alleging that a salesperson “made fraudulent statements that induced the plaintiff to sign the contract to the plaintiff’s detriment.” The salesperson then filed a motion to dismiss the complaint for failure to state a claim upon which relief can be granted.

How should the court rule?

A Deny the motion, because the complaint stated a possible claim for relief.

B Deny the motion, because the complaint stated a plausible claim for relief.

C Grant the motion, because the complaint did not state the claim for relief with particularity.

D Grant the motion, because the complaint did not state the claim for relief with probability.

A

The court should grant the motion to dismiss for failure to state a claim upon which relief can be granted. The federal pleading rules generally require the pleader to assert short and plain statements in the complaint to put the other side on notice of the claim being asserted; detailed assertions of facts underlying the claim are generally not required. However, there are certain special pleading rules that require a party to state more detail under special circumstances, including claims that assert fraud or mistake. Under such special circumstances the federal rules specifically require that a plaintiff assert the claim for relief with particularity. Here, choice (C) is the best answer because it is the only choice to correctly state the rule. The salesperson’s motion to dismiss should be granted in this case because the plaintiff’s complaint fails to state the claim with particularity. Choices (A), (B), and (D) all misstate the special pleadings rule that applies to complaints asserting fraud and mistake.

141
Q

A federal court will not hear a case unless there is a real, live controversy at all stages of the proceeding, not merely when the case is filed.

What about class action?

A

However, a class action is not moot, and the class representative may continue to pursue it—even if the representative’s own controversy has become moot—because the claims of others in the class are still viable.

142
Q

A storeowner properly filed a complaint for breach of contract against a food distributor in federal district court. After the food distributor timely answered the complaint, the parties proceeded through discovery, which lasted over 14 months and cost the parties over $200,000 in attorneys’ fees and related costs. At the final pretrial conference, the presiding judge indicated that he did not think much of the merits of the storeowner’s claims. As a result, the storeowner wants to dismiss this case and refile in a different federal court to get a more sympathetic judge.

How may the storeowner try to achieve this goal?

A File a motion for voluntary dismissal without prejudice, and it will likely be granted on the grounds that the judge is biased against the storeowner’s case.

B File a motion for voluntary dismissal without prejudice, but it will likely be denied due to the time and money already invested in the case by the parties and the court.

C File a notice of dismissal with the federal district court, and it will likely be granted because the Federal Rules of Civil Procedure give a plaintiff the right to unilaterally dismiss an action once without prejudice.

D File a motion for voluntary dismissal without prejudice, and it will likely be granted because courts are required to freely grant such motions to further judicial economy.

A

The storeowner can give up his case voluntarily by way of voluntary dismissal.

If the defendant has answered or filed a motion for summary judgment or there was a previous dismissal, the plaintiff must file a motion for voluntary dismissal by leave of the court, and the court has the discretion to grant dismissal on such terms and conditions as the court deems proper.

Here, because the distributor answered the complaint, this motion for voluntary dismissal with leave of the court was a proper vehicle to attempt to achieve the storeowner’s goal; however, due to the excessive time (14 months) and expense (over $200,000), it is unlikely that the court would deem these conditions proper to grant such a dismissal. (A) is wrong because, although it correctly states that the proper motion to file is a motion for voluntary dismissal without prejudice, the outcome based on the rationale is highly unlikely without other evidence. Judges are free to comment regarding their opinion of the evidence presented, and without more, there is no evidence that such a comment rises to the level of the judge being biased sufficiently to grant such a motion. (C) is wrong because it is not procedurally correct. A notice of dismissal may be filed by a plaintiff who wishes to voluntarily dismiss the case without leave of the court; however, it is only proper if the defendant has not answered or filed a motion for summary judgment. Here, the distributor answered the complaint. Therefore, the notice of dismissal is improper. (D) is wrong because, although like choice (B), it correctly states that the proper motion to file is a motion for voluntary dismissal without prejudice, the outcome based on the rationale is incorrect. There is no such rule that courts are required to freely grant such motions to further judicial economy. In fact this statement is counterintuitive as to do so would cost more judicial time and resources

143
Q

Is joint totfeasor an indispensable party?

A

Supreme Court has held that a joint tortfeasor subject to joint and several liability is not a person needed for just adjudication

144
Q

A utility company constructed a building costing approximately $2 million that encroached on a rancher’s property. The rancher is suing the company in federal district court to force the company to remove the office building. In the same action, the rancher is asking for $200,000 in damages incurred because of the trespass. The court has diversity of citizenship jurisdiction.

Is the company entitled to a jury trial?

A Yes, for all issues.

B Yes, but only for issues involving the trespass action.

C Yes, but only for issues involving the action to require the company to move its building.

D No, because the equitable issue predominates.

A

The company is entitled to a jury trial only for issues involving the trespass action. In this case, both legal and equitable issues exist. The defendant is entitled to a jury trial on the legal issues in the case, even though the equitable issue of whether an injunction mandating removal of the building should be issued clearly predominates. Hence, (D) is incorrect. (A) and (C) are incorrect because, as stated, the issue regarding removal of the building is equitable, and thus there is no right to answer it to a jury.

145
Q

In a drug raid, police in a city searched 25 apartments selected at random in a 300-unit housing project. In a class action approved by the federal court, the 25 tenants sued the city for violation of their constitutional rights. The tenant named as class representative gave notice to all unnamed class members, including another tenant who decided not to opt out. The class action was then certified as a “common question” type. After negotiating with the class representative, the city police agreed to pay each tenant $500 and to conduct no further raids without proper warrants. The representative and the city signed a settlement agreement and a stipulation of dismissal of the class action. The other tenant objects to the amount of damages he is to receive and would rather opt out now and proceed on his own.

May the tenant opt out now?

A Yes, because a class member of a “common question” class action always has the right to opt out.

B Yes, if the court allows the tenant a second opportunity to opt out.

C No, because the tenant did not opt out after receiving notice of the class action.

D No, if class certification was proper.

A

The tenant may opt out if the court allows class members a second opportunity to opt out.

In a “common question” class action, a judge may refuse to approve a settlement of a class action unless the class members are given a second opportunity to opt out. [Fed. R. Civ. P. 23(e)]

(A) is incorrect because there are only two opt-out periods, with the initial notice and with the settlement agreement, and the latter only applies if the judge requires it. (C) is incorrect and reflects a prior version of Rule 23. (D) is incorrect for the reasons stated above.

146
Q

Objections to venue may be raised in

A

a defendant’s answer if the defendant did not assert a Rule 12(b) pre-answer motion

147
Q

Pursuant to Fed. R. Civ. P. 33, each party is limited to … interrogatories.

Pursuant to Fed. R. Civ. P. 30, a party is entitled to … depositions as a matter of right.

A

25

10

148
Q

Joining unrelated claims

A

Pursuant to Fed. R. Civ. P. 42(b), the court may join unrelated claims between a party, but may decline to do so when doing so would cause jury confusion or some other prejudice.

149
Q

Is a motion to increase a jury’s award of damages perimissible?

A

A motion to increase a jury’s award of damages is a motion for additur. Additur is unconstitutional in federal court.

150
Q

If an action was originally brought in an improper venue, i.e., one that does not have jurisdiction over the defendant, then the action never belonged in that venue.

A

Therefore, a transferee venue will apply its own conflict of law principles to determine the applicable substantive law.

151
Q

Pursuant to Fed. R. Civ. P. 30(b)(6), when a corporation is named as a deponent, it must designate an individual who will testify about facts reasonably known to the organization.

A

The CFO failed to discover what was reasonably known to the organization, thus the corporation failed to provide the required disclosure.

The proper motion for the plaintiffs to make in this situation is a motion to compel a discovery response.

A motion to compel disclosure is the applicable motion when a party has failed to make a disclosure required by Fed. R. Civ. P. 26(a). That is inapplicable to this situation which deals with the responses required in a deposition.

152
Q

Remand

A

If a claim is filed in state court that could have originally been filed in federal court, the defendant(s) have the option to remove the case to federal court within 30 days of service of the initial pleading. If a new defendant is added to the action, that new defendant has 30 days to decide whether or not to remove.

However, there is an exception to the right of removal. Where the federal court would have subject-matter jurisdiction based on diversity and one of the defendants is a citizen of the forum state, the claim cannot be removed to federal court. This is known as the home-state defendant rule.

153
Q

A federal court must have subject-matter jurisdiction over each cause of action alleged in the complaint. Nevertheless, claims that do not provide subject-matter jurisdiction may be combined with appropriate claims through the court’s supplemental jurisdiction, provided that they arise out of the same case or controversy [28 U.S.C. Sec. 1367]. Claims arise out of the same case or controversy when they derive from:

A

(1) the same transaction or occurrence; or (2) a common nucleus of operative facts.

154
Q

A court may impose sanctions on a party that fails to respond to a request for waiver, but will not do so when there was “good cause” for failing to respond, i.e., where the party did not receive the waiver.

A

Belief that the court did not have jurisdiction, however, , no matter how well founded (i.e. exagerated amount of damages), does not qualify as good cause, and therefore the general counsel’s reason for not responding will not prevent the court from imposing sanctions.

155
Q

Pursuant to Fed. R. Civ. P. 65(d)(2), a preliminary injunction will apply to

A

a party, as well as that party’s officers, servants, agents, employees, and attorneys.

156
Q

Motions for judgment as a matter of law are an option available in civil trials to both the plaintiff and the defendant.

A

In a criminal case, however, the judge can only grant judgment in favor of the defendant. The judge cannot grant judgment finding the defendant guilty, because the defendant is entitled under the Constitution to a full and complete trial.

157
Q

According to Rule 48(a), a jury must begin with at least six and no more than 12 members, and each juror must participate in the verdict unless excused under Rule 47(c).

A

dditionally, unless the parties stipulate otherwise, the verdict must be unanimous and must be returned by a jury of at least six members [Fed. R. Civ. P. 48(b)].

Here, the verdict was unanimous, but because another juror became ill and was dismissed after deliberations began but before the verdict was returned, the jury that returned the verdict was comprised of only five members, less than the minimum. Because the facts do not indicate that the parties stipulated that this would be acceptable, the company’s motion for a new trial should be granted. (A) is incorrect. Just because the timing requirement was satisfied does not mean that the motion should be granted. The motion should be granted because there is a valid reason (violation of the law), rather than simply because it was made within the time frame for a validly filed motion.

158
Q

A single man with a life insurance policy that pays his designated beneficiary $70,000 upon his death was killed in a car accident. His former girlfriend, a resident of State A, was named as beneficiary, but his mother, a resident of State B, also filed a claim for the life insurance proceeds. The insurance company, a State C corporation having its principal place of business in State B, filed an interpleader action in federal court to protect itself from potentially multiple and inconsistent claims.

May the insurance company bring the interpleader action in federal court?

A No, because the amount in controversy does not exceed $75,000, and there is no diversity between the insurance company and one of the claimants.

B No, because there is no federal court where the insurance company will be able to establish personal jurisdiction over both claimants.

C No, because the insurance company is not diverse from one of the claimants.

D Yes, because the amount in controversy is $500 or more, and both claimants are diverse from one another.

A

The insurance company can bring the interpleader action in federal court.

The Federal Interpleader Act provides special rules for dealing with subject matter jurisdiction, personal jurisdiction, and venue.

Under the act, interpleader can be brought if any two claimants are citizens of different states and the amount in controversy is $500 or more. Complete diversity is not required.

Here, the mother is a State B resident, and the former girlfriend is a State A resident. Thus, both claimants are diverse from one another, and the $70,000 amount in controversy exceeds the $500 minimum.

159
Q

A plant nursery sued the owner of a nearby factory in federal court, alleging that the factory was emitting toxic fumes that were harming the nursery’s plants. The nursery sought an injunction and damages. During the hearing for the preliminary injunction, the factory’s owner correctly pointed out that there was conflicting federal precedent as to whether injunctive relief was appropriate under the facts. Nonetheless, the court granted a preliminary injunction against the factory, ordering it to stop operations until a final judgment was reached in the case. It further refused to certify the issue for appeal. The damages issue has not yet been addressed. The factory owner files an appeal against the injunction.

Can the court of appeals hear the factory owner’s appeal?

A Yes, because there is conflicting federal precedent whether an injunction is appropriate under the facts.

B Yes, because injunctions are reviewable on appeal before there is a final judgment.

C No, because the granting of a preliminary injunction is not a final order.

D No, because the judge did not certify that the order has a controlling question of law as to which there is a substantial ground for difference of opinion and that an immediate appeal from that order may materially advance the ultimate termination of the litigation.

A

The court of appeals can hear the factory owner’s appeal because injunctions are reviewable before there is a final judgment. Generally, only final orders are reviewable, but certain interlocutory orders, including those involving injunctions, are immediately reviewable.

(A) and (D) are incorrect for similar reasoning. Under the Interlocutory Appeals Act, a trial court judge may certify that the order has a controlling question of law as to which there is a substantial ground for difference of opinion and that an immediate appeal from that order may materially advance the ultimate termination of litigation, and the appellate court may, as a discretionary matter, hear the appeal. However, that Act also allows appeals of orders relating to injunctions regardless of whether there is unclear case law or whether the judge has certified the issue. Thus, is does not matter that there is conflicting federal precedent or that the trial judge has not certified the order as described above. (C) is incorrect because a final judgment is not required when a party appeals an order granting, modifying, or denying an injunction.

160
Q

A homeowner from State A hired a contractor from State B to build a vacation home for her in State C. The parties signed the contract in State A. The contractor breached the contract, and the homeowner sued the contractor in a court of State A, seeking damages of $100,000. The contractor removed the case to the federal court for State A. The homeowner promptly moved to remand the case to state court, arguing that venue was improper.

Which of the following facts is most relevant to the court’s decision on the homeowner’s motion?

A The contract was signed in State A.

B The contractor resides in State B.

C The homeowner commenced the action in a State A court.

D The vacation home was to be built in State C.

A

(C) The commencement of the action in State A is most relevant. When a case is removed from state court to federal court, venue is set in the federal district court that embraces the state court in which the action was pending, making the federal district court of State A the only appropriate venue choice. Thus, (C) is the correct answer because the most (indeed, only) relevant fact is the fact that the action was commenced in a court of State A. Unlike cases commenced in federal court, in cases removed to federal court, the residence of the defendant contractor (answer choice (B)) and the place where a substantial part of events relating to the claim occurred (answer choices (A) and (D)) are irrelevant.

161
Q

A builder from State A sued a homeowner from State B for breach of contract in federal court, alleging that the homeowner failed to pay the second half of the agreed-upon price for completion of construction on a house. The process server attempted to serve the summons and complaint on the homeowner at the house, but after two failed attempts, instead served it on an elderly next-door neighbor, who was out in the front yard and volunteered to accept it on the homeowner’s behalf. The neighbor gave the homeowner the summons and complaint later that day.

The homeowner then filed a motion to dismiss for insufficient process. Assume that the requirements for service of process in both states are identical to the requirements of the Federal Rules of Civil Procedure.

Is the court likely to dismiss the action for insufficient process?

A Yes, because the homeowner did not designate the neighbor as an authorized agent to receive service of process.

B Yes, because the neighbor is not considered one residing at the homeowner’s place of abode, even though the neighbor is of suitable age.

C No, because the homeowner filed the wrong motion.

D No, because the homeowner has no meaningful defense for insufficient service of process since she had actual notice.

A

The court will not likely dismiss the action.

Insufficient process refers to defects in the documents and their content, whereas insufficient service of process refers to the manner in which the documents were presented, delivered and/or served to the defendant. The key to this answer is reading comprehension. The facts state that the homeowner filed a “motion to dismiss for insufficient process,” not “insufficient service of process,” which are two separate grounds for dismissal. [See Fed. R. Civ. P. 12(b)(4) and (5), respectively] (A), (B) and (D) are all wrong because they all relate to issues regarding insufficient service of process, which is not relevant to the call of the question. (C) is correct, because the homeowner filed the wrong motion. The homeowner should have filed a motion to dismiss based on insufficient service of process. Generally speaking, service of process is sufficient if it is made by: (i) personal service, (ii) service left at the defendant’s usual place of abode with one of suitable age and discretion, or (iii) service upon an authorized agent of the defendant. Alternatively, service may be made under state rules or by mail under the waiver of service provision [see Fed. R. Civ. P. 4(d)].

162
Q

A State A citizen and a State B citizen were in a car accident in State A. The State A citizen filed a negligence action in a State A state court seeking $200,000 in damages. The State B citizen filed an answer and four months later filed a notice of removal, removing the action to federal court. Two months after the State B citizen filed and served the notice of removal, the State A citizen filed in the federal court a motion to remand the case back to state court.

Should the federal court remand the action to state court?

A No, because the State B citizen was not a citizen of the forum state.

B No, because the motion to remand the action to state court was untimely.

C Yes, because the notice of removal was untimely.

D Yes, because federal courts do not have subject matter jurisdiction over state law negligence actions.

A

Ms. Removal is 30 days old.

The federal court should not remand the action to state court. A defendant may remove an action that could have originally been brought in the federal courts.

Generally, a defendant must file a notice of removal within 30 days after receipt by or service on the defendant of the initial pleading or summons (when he learns the case is removable).

A defendant may not remove a case based solely on diversity if he is a citizen of the forum state; however, that restriction is not jurisdictional but rather procedural. Once removed, a case must be remanded back to state court if the federal court lacks subject matter jurisdiction.

A motion to remand based on procedural defects in removal must be filed within 30 days of removal.

Here, the federal court has diversity of citizenship jurisdiction over the action. The plaintiff is from State A and the defendant is from State B (and thus not a citizen of the forum state), and the amount in controversy is $200,000. Although the notice of removal was untimely, the motion to remand was also untimely. *A motion to remand on grounds other than a lack of subject matter jurisdiction (e.g., the notice of removal was untimely filed) must be filed within 30 days of removal. The failure to timely remand the case on procedural defects is fatal.* Thus, (B) is correct. (A) is incorrect. Although a defendant may not remove a case based solely on diversity when he is a citizen of the forum state, that fact does not address whether the motion to remove or remand was timely. Had the State A citizen timely filed a motion to remand, the motion to remand would have been granted. (C) is incorrect. Although it is true that the notice of removal was not timely, the plaintiff must object to any defects in removal by filing a motion to remand with the federal court within 30 days. Here, the State A citizen failed to do so. (D) is incorrect because state law claims may be heard in federal court under the federal court’s diversity of citizenship jurisdiction.

163
Q

An employee properly sued her employer for wrongful discharge in federal court. During discovery, the employee served the employer with a discovery request for information regarding all employment termination over the previous 15 years, regardless of the position. The employer objected, and the employee filed a motion to compel the requested discovery. The court denied the motion to compel, and the employee wants to file an immediate appeal to review this decision.

Does the employee have a right to an immediate appeal?

A Yes, because the trial court’s order is a “collateral order” that is immediately appealable under the collateral order rule.

B Yes, because the trial court’s order is a “final judgment” that is immediately appealable.

C Yes, because the plaintiff may obtain immediate appellate review of the trial court’s order through a writ of mandamus.

D No, because the trial court’s order is an “interlocutory order” that can be reviewed prior to final judgment only if the trial court certifies it for immediate appeal and the appellate court, in its discretion, agrees to hear the appeal.

A

The employee does not have the right to an immediate appeal.

Interlocutory orders are the rulings that trial judges make during the course of pretrial proceedings and trials that do not completely resolve the case. Therefore, they are not final. As a result, interlocutory orders are typically not immediately reviewable on appeal until a final order is made, unless they meet one of the exceptions permitting an appeal as of right (i.e., orders granting injunctions; orders appointing a receiver; orders in admiralty cases finding liability but leaving damages to be assessed later; patent infringement orders where only an accounting is ordered; and orders affecting or changing possession of property).

The Interlocutory Appeals Act also may permit a review of an interlocutory order, but it is discretionary, and may be available only when

(i) the trial judge certifies that the interlocutory order involves a controlling question of law, as to which there is substantial ground for difference of opinion, and immediate appeal from the order may materially advance the ultimate termination of the litigation; and

(ii) the court of appeals then agrees to allow the appeal.

A party obtaining such a certificate from the trial judge must, within 10 days, apply to the court of appeals, where two out of three judges must agree to hear the appeal.

Here, the trial court’s denial of the employee’s pretrial motion to compel discovery is an interlocutory order. As such, there is no immediate right to appeal. Choice (D) correctly states the rule under the Interlocutory Appeals Act. The Act only offers a discretionary review; it is not reviewable by right. (A) is wrong because it incorrectly classifies the order as a collateral order. The collateral order rule may permit a review if the claim or issue is separable from and collateral to the main suit and is too important to require deferring appellate review. The order then may be classified as a judgment in a separate (“collateral”) proceeding. Here, the order relates to a discovery request involving the main issue—wrongful discharge. Therefore, it is not a collateral order. (B) is wrong because, as stated above, this is an interlocutory order, not a final order. A final order is one that disposes of the whole case on its merits, by rendering final judgment not only as to all the parties, but as to all causes of action involved. Here, there has not even been a trial yet, and the pretrial order only addresses a motion to compel discovery. As such, it is an interlocutory order. (C) is wrong because writs of mandamus are only permitted in exceptional cases. Mandamus commands a trial judge to act, and prohibition commands the judge to refrain from acting. The writs are available only if an appeal will be insufficient to correct a problem and the trial court’s actions constitute a serious abuse of power that must be immediately corrected. The facts in this question clearly do not support such exceptional circumstances.

164
Q

A transportation company from State A sued a tourism company from State B in federal court for breach of contract seeking $76,000 in damages. The transportation company filed an appropriate demand for a jury trial, and both parties stipulated that the jury would consist of 10 people. Following the trial and deliberations, nine of the jurors voted in favor of the plaintiff for $29,000 in damages, but one juror voted for the defendant. The defendant appealed.

On which of the following grounds would the defendant most likely prevail on appeal?

A The court lacked subject matter jurisdiction because the verdict did not exceed $75,000.

B The court lacked personal jurisdiction.

C The ruling was defective because the jury’s vote was not unanimous.

D The ruling was defective because it was rendered by fewer than 12 jurors.

A

The ruling was defective because the jury’s vote was not unanimous. In federal civil cases, a verdict must be unanimous unless the parties agree to the contrary. Here, there are no facts indicating that the parties agreed to less than a unanimous vote. Therefore, the ruling is defective because it was not unanimous.

165
Q

A homeowner filed a federal diversity action against the manufacturer of a pressure cooker for a products liability action based on state law, alleging that she was severely burned because the top flew off the pressure cooker during operation. Under the state law, a jury verdict does not have to be unanimous. However, a specific rule under the Federal Rules of Civil Procedure directly conflicts with the state law and provides that jury verdicts in federal court must be unanimous.

Which statement correctly presents what law the federal judge should apply regarding the jury verdict?

A The state law will apply, because a federal district court with subject matter jurisdiction based on diversity of citizenship must apply state law when there is a conflict.

B The state law will apply, because applying the specific Federal Rules of Civil Procedure may change the outcome of the case.

C The specific Federal Rule of Civil Procedure will apply as long as both rules are consistent with the Rules Enabling Act and are not unconstitutional.

D The specific Federal Rule of Civil Procedure will apply, because, on balance, the federal judicial system has the greater interest in having its rules applied.

A

The Federal Rule will apply.

Under the Erie doctrine, although the federal court generally applies the substantive law of the state in which it is sitting, when a state law-based claim is brought in federal court based on diversity of citizenship and a specific federal statute or a Federal Rule of Civil Procedure applies, the federal court must apply federal procedural law as long as the federal rule is rationally deemed to be procedural and does not abridge, enlarge, or modify any substantive rights.

Here, there is a specific federal procedural rule [Rule 48] under the Federal Rules of Civil Procedure that requires jury verdicts to be unanimous. This rule is procedural because it is designed to deal with what happens in the process of litigation, as opposed to dealing with the rights of the parties outside of court, which would be deemed a substantive rule. Moreover, the federal rule directly conflicts with the state law. Therefore, the specific Federal Rule of Civil Procedure will be applied in federal court, as long as it is consistent with the Rules Enabling Act and is not unconstitutional.

(A) is wrong because it incorrectly states that the state law will apply and gives an incorrect statement of the law. State law does not always govern when there is a state-based claim filed in federal court based on diversity, as stated above.

(B) is wrong because it incorrectly states that the state law will be followed, and it incorrectly references the “outcome determinative” test. This test was set forth by the Supreme Court to use when the law is unclear regarding whether an issue is substantive or procedural for Erie purposes and holds that an issue is substantive if it substantially affects the outcome of the case. Here, there is a Federal Rule of Civil Procedure that is directly on point. As such, the test does not apply to this situation.

(D) is wrong because it incorrectly references the “balance of interests” test, which is another test the Supreme Court set forth to use when the law is unclear regarding whether an issue is substantive or procedural for Erie purposes. Under this test, the federal court is to weigh whether the state or federal judicial system has the greater interest in having its rule applied. Again, here there is a Federal Rule of Civil Procedure that is directly on point. As such, the test does not apply to this situation.

166
Q

The plaintiff sued the defendant in federal court for breach of contract. The case went to trial, and the jury found in favor of the plaintiff and awarded her $125,000. Judgment was entered on June 1. On June 10, the defendant filed a motion for a new trial. On June 18, the plaintiff files to enforce the judgment. The court has not issued any orders since the final judgment on June 1.

May the plaintiff enforce the judgment?

A Yes, because judgments are enforceable as soon as they have been entered.

B Yes, because judgments are enforceable during pendency of post-trial motions unless the court otherwise orders.

C No, because judgments are not enforceable until 28 days after entry.

D No, because judgments cannot be enforced while a post-trial motion is pending.

A

The plaintiff may enforce the judgment because judgments are enforceable during pendency of post-trial motions unless the court otherwise orders and on such conditions for the security of the adverse party as are proper.

Here, the court has not ordered a stay on enforcement; therefore, the plaintiff can enforce the judgment. (D) is therefore incorrect. (A) and (C) are incorrect because execution on judgments is not allowed for 14 days after entry except in the case of injunctions or receiverships, which are immediately enforceable unless otherwise ordered by the court.

167
Q

A plaintiff who was a citizen of State A was traveling to adjoining State B to visit his relatives. While still in State A, the plaintiff’s auto was struck in the rear by a vehicle driven by the defendant, a citizen of State B. The plaintiff suffered personal injuries and damage to his vehicle amounting to approximately $90,000. The plaintiff filed suit in the federal district court for State A and obtained proper service of process on the defendant. Under the laws of State A, the driver of a vehicle that strikes another vehicle in the rear is presumed to have acted negligently, regardless of the surrounding circumstances. Neither the law of State B nor the federal statutes or case law has adopted such a rule.

Should the court apply the presumption in question?

A No, because federal law does not recognize such a presumption.

B No, because the law of State B does not recognize such a presumption.

C Yes, because in a diversity case a federal court applies the substantive and procedural laws of the state in which it sits.

D Yes, because the presumption at issue operates upon elements of the prima facie case.

A

The court should apply the presumption. Under the Erie doctrine, in a case based on diversity of citizenship, the federal court must apply the substantive law of the state in which the court sits. However, the court will apply federal law to procedural issues. Federal Rule of Evidence 302, which follows the Erie doctrine, provides that application of state law is appropriate only when the presumption operates on a substantive element of a claim or defense. The presumption at issue here, by presuming negligence on the part of a driver who strikes another vehicle in the rear, impacts on the prima facie case elements of duty and breach of duty. Matters involving elements of a prima facie case are substantive in nature; thus, state law applies to such matters. Consequently, the presumption of negligence recognized by State A should be applied by the court on this issue, and (A) is therefore incorrect. (B) is incorrect because it is immaterial whether State B recognizes the presumption. (C) is incorrect because it states that a federal court applies both the substantive and procedural law of the state in which the federal court sits. It applies only substantive state law.

168
Q

A homeowner sued a contractor in federal court for damages resulting from an unsuccessful roof repair. However, the homeowner failed to attend status conferences and failed generally to prosecute the action. As a result, the court dismissed the case with prejudice for want of prosecution. The homeowner then filed her complaint in a different federal court. In his answer the contractor asserts the affirmative defense of claim preclusion.

Should the court dismiss the case?

A No, because the merits of the case were not determined.

B No, because the court erred in designating the dismissal as one with prejudice.

C Yes, because the dismissal was with prejudice.

D Yes, because dismissals for want of prosecution are on the merits.

A

The court should dismiss the case.

For claim preclusion to apply,

(i) the earlier judgment must be a valid, final judgment on the merits;
(ii) the cases must be brought by the same claimant against the same defendant; and
(iii) the same cause of action must be involved in the later suit.

Under Federal Rule of Civil Procedure 41, dismissals for lack of prosecution are with prejudice and operate as an adjudication on the merits unless the court designates otherwise. As a result, the judgment has claim preclusion effect on subsequent cases between the parties. (A) is incorrect. Although the factual merits of the case were not determined, Rule 41 dictates that the judgment is on the merits unless the court states otherwise. (B) is factually incorrect, as the court complied with Rule 41. (D) is incorrect as too broad. The court may designate a dismissal for want of prosecution as an adjudication on the merits but is not required to do so. Here, the court could have designated the dismissal as one without prejudice, meaning that the plaintiff could bring her action in another court.

169
Q

An entry of default may be set aside for …

A

… “good cause shown.”

Although not specifically required by the Federal Rules, a majority of courts also will require some showing of a meritorious defense.

170
Q

A customer slipped and fell in a store and was severly injured. As required by state regulation, the store’s manager conducted an investigation into the cause of the accident and filed the report with the appropriate state agency. The customer subsequently retained an attorney, who filed a civil action against the store in federal district court. The complaint alleged that the store negligently left a spill on the floor of the store, which caused the customer’s fall. The customer’s attorney served on the store a request for production of documents, which included a request for all documents and reports prepared by the store that relate to the customer’s fall and injury.

If the store believes that the report prepared by the store manager is protected from discovery under the work product doctrine and the store wants to withhold disclosure of the report, what should the store do?

A Produce all the documents that respond to the request that are not protected and simply not include the report in the documents produced.

B File a motion for a protective order with the court before its response to the request for documents is due, to avoid waiving any protection from discovery that it might have had.

C Describe the nature of the report so that the customer and her lawyer can assess the validity of the store’s claim that the report is protected from discovery.

D File the report with the court under seal so that the court may assess whether it is protected from discovery.

A

The store should describe the nature of the report so that the customer and her lawyer can assess the store’s claim that the report is protected from discovery.

When a party claims that certain discoverable information is privileged trial preparation material, it must disclose the existence of the material in sufficient detail to the opposing party so that the opposing party may assess the claim of privilege.

(A) is incorrect because a party must disclose the existence of material it claims is protected by privilege. (B) and (D) are incorrect because they do not describe the proper procedure for claiming that material is privileged work product.

171
Q

If a witness travels to another jurisdiction solely to be a witness in a court action, the witness is immune from service of process. Therefore, the witness may not be served with process in person while in the other jurisdiction for court purposes. The immunity rule applies to witnesses, parties, and attorneys alike.

A

Thus, it is not correct that he is immune from process as long as he is only a witness and not a party. Immunity from being a witness is a separate concept from being fraudulently induced to come into a jurisdiction for purposes of serving process and obtaining personal jurisdiction. Participation in a judicial proceeding is grounds for immunity and does not require fraud on the part of the serving party. Thus, the choice providing immunity only if he were fraudulently served with a subpoena to be a witness is incorrect.

172
Q

A court may grant, in its discretion, an ex parte temporary restraining order if the moving party gives specific facts in an affidavit or in the verified complaint to establish that immediate and irreparable injury will result to the moving party before the adverse party can be heard in opposition. The other requirements are that the moving party must

A

(i) certify in writing all efforts she made to give notice of the hearing to the adverse party and the reasons why notice should not be required; and
(ii) provide some security, the amount of which is determined by the court, to pay for any costs and damages incurred by the adverse party if he was wrongfully enjoined or restrained.

The United States, its officers, and its agencies are not required to give security.

Preliminary injunctions, summary judgments, and judgments as a matter of law require notice to the other party in order to be issued.

173
Q

The Seventh Amendment provides the right to a jury trial in federal courts for the determination of facts in

A

all suits at common law where the amount in controversy exceeds $20.

174
Q

The Supreme Court has held that when legal and equitable claims are joined in one action,

A

the legal claim should be tried first to the jury and then the equitable claim should be decided by the court. The jury’s fact determinations bind the judge.

175
Q

A motion for relief from judgment because the judgment is void must be brought within a reasonable time, but there is no outermost deadline. When relief is based on

(i) mistake, inadvertence, surprise, or excusable neglect;
(ii) newly discovered evidence that, by due diligence, could not have been discovered in time to move for a new trial; or
(iii) fraud, misrepresentation, or other misconduct of an adverse party, the motion must be brought within

A

a reasonable time not to exceed one year.

176
Q

Sc may hear cases

A

The Supreme Court may hear some appeals from state courts and some appeals directly from federal district courts. The Supreme Court hears, on direct appeal, any order granting or denying an injunction in any proceeding required to be heard by a three-judge district panel. It also may hear by discretionary writ of certiorari final judgments of the highest court of a state if: (i) the validity of a treaty or federal statute is drawn into question; (ii) the validity of a state statute is drawn into question on the ground that it is repugnant to the federal Constitution or to a treaty or federal statute; or (iii) any title, right, privilege, or immunity is claimed under the federal Constitution or treaty or federal statute.

177
Q

Under the Full Faith and Credit Clause

A

a state court generally must recognize the judgments of a sister state and federal courts generally must recognize judgments of state courts. Recognition of judgments is required between state courts, between state and federal courts, and between federal courts.

178
Q

If a party fails to support an assertion of fact or fails to properly address another party’s assertion of fact, the court may consider the fact undisputed for purposes of the motion, grant summary judgment if appropriate, give the party an opportunity to address the fact, or issue any other appropriate order.

A

Here, the plaintiff came forward with affidavits supporting his contention that the light was red when the defendant entered the intersection. The defendant here must counter those affidavits with her own or risk summary judgment being entered against her.

179
Q

The Seventh Amendment preserves the right to a jury trial for both the plaintiff and defendant for civil actions in federal court of facts in all “suits of common law” where the amount in controversy exceeds $20. The determination is historical and turns initially on whether the claim or relief was available at law or in equity in 1791. However, the Supreme Court has demonstrated a clear preference for jury trials in doubtful cases and has held that, if damages are claimed as part of an action seeking an injunction, the defendant cannot be denied a jury on the damages issues on the ground that they are “incidental” to the equitable relief.

A

As such, when a case presents both legal and equitable claims, the right to a jury is preserved for all legal claims, but not for equitable claims. Instead, the legal claim should be tried first to the jury, and the equitable claim will go to the court (the jury’s finding on fact issues will bind the court in the equitable claims).

Here, the call of the question asks if the former manager can demand a jury trial for both claims. Under the Seventh Amendment, he only has a right to a jury trial on the legal claim for money damages, not the equitable claim for the injunction. Instead, the court will hear the equitable claim for the injunction, but will be bound by any of the jury’s factual decisions that relate to the equitable claim.

180
Q

An elderly woman entered into a contract with a company that was in the business of providing home care services. Believing that she had been duped by representatives of the company, the woman commenced an action in federal court, properly based on diversity, seeking rescission of the contract. The company answered, denying the principal allegations of the woman’s complaint and asserting a counterclaim against the woman for breach of contract. In addition, the company timely served a demand for a jury trial. The woman did not. Which statement best describes the roles of the judge and jury as finders of fact in the trial of the parties’ claims?

A

The jury will first determine the issues relating to the breach of contract claim, and the judge will determine the issues relating to the rescission claim that have not already been resolved by the jury. à If legal and equitable claims are joined in one action involving common fact issues, the legal claim is tried first before the jury, and then the equitable claim is tried to the court. The jury’s finding on fact issues will bind the court in the equitable claim. (A) is incorrect because all fact issues relating to the company’s legal claim must first be determined by the jury. Only then may the judge try the woman’s equitable claim. (C) and (D) are both incorrect because neither the jury nor judge acts as the sole finder of fact when the case has legal claims and equitable claims, and a jury has been demanded on the legal claims.

181
Q

. If the plaintiff bases the motion to remand on a defect other than subject matter jurisdiction (i.e., a defect in the removal procedure), the motion to remand must be brought within 30 days of removal.

A

. There is no such time limit for a lack of subject matter jurisdiction. The court must remand whenever it is shown that there is no subject matter jurisdiction.

182
Q

Under the Erie doctrine, when a state law-based claim is brought in federal court based on diversity of citizenship, the federal court generally applies the substantive law of the state in which it is sitting. However, where a specific federal statute or the Federal Rules of Civil Procedure are on point, the federal court must apply federal procedural law as long as the federal rule is valid.

A

Under the Rules Enabling Act, a Federal Rule is valid if it deals with “practice or procedure” and does not “abridge, enlarge, or modify” a substantive right.

Here, there is a specific federal procedural rule that is on point [Fed. R. Civ. P. 48], which requires jury verdicts to be unanimous, unless the parties agree otherwise. Since there is no evidence of agreement, the federal procedural rule will apply, and the motion should be denied.

183
Q

Under issue preclusion (collateral estoppel),

A

a judgment binds the plaintiff or defendant (or their privies) in subsequent actions on different causes of action between them (or their privies) as to issues actually litigated and essential to the judgment in the first action.

Issue preclusion focuses on something relatively narrow–an issue that was litigated and determined in the first case, and that is relevant in a second case. With issue preclusion, the issue is deemed established in the second case without need to proffer evidence on it.

i.e.

Here, the car driver was a party to the first lawsuit and, after both parties “vigorously litigated” the issues of negligence, the jury determined that the car driver was negligent in the first case. Since the car driver filed a negligence claim against the truck driver claiming that he ran the red light in the first case, and the truck driver defended on the grounds that the car driver was contributorily negligent for speeding, the issue of who was negligent was essential to the judgment in the first action.

Therefore, the court will preclude the car driver from relitigating whether she was negligent in the claim asserted against her by the passenger. For these same reasons, (A) is wrong.

(B) is wrong because the car driver’s due process rights are not violated. Due process requires that issue preclusion may be asserted only against someone who was a party (or in privity with a party) to the previous case (the case in which the issue was actually litigated and determined). Here, due process is satisfied because the car driver was a party in the previous claim and vigorously litigated the claim. Note that the car driver may be able to assert the traditional “mutuality” rule to prevent the passenger from asserting issue preclusion, but this is not what was asserted in choice (B); due process was.

Under the traditional “mutuality” rule, only someone who was a party (or in privity with a party) in the previous case can use issue preclusion. However, this requirement is not imposed by due process and has been subject to modification in certain circumstances to allow nonparties to take advantage of a prior judgment. In fact, although courts have been very reluctant to permit a nonparty to use issue preclusion to aid him offensively (as a plaintiff) to obtain relief, the Supreme Court has upheld its use offensively by a nonparty where it was fair and equitable to do so. [Parklane Hosiery Co. v. Shore, 439 U.S. 322 (1979)]

(D) is wrong because it incorrectly states that the claim will be precluded by claim preclusion (res judicata). Claim preclusion focuses on something relatively large–the scope of a “cause of action.” If it applies, the result is usually to bar the same party in the first case from asserting a second case against the other party in the first case. Issue preclusion, in contrast, focuses on something relatively narrow–an issue that was litigated and determined in the first case and that is relevant in a second case.

184
Q

On motion and just terms, the court may relieve a party from a final judgment or order on the following grounds:

(i) mistake, inadvertence, surprise, or excusable neglect;

(ii) newly discovered evidence that by due diligence could not have been discovered in time to move for a new trial;

(iii) fraud, misrepresentation, or other misconduct of an adverse party;

(iv) the judgment is void;

(v) the judgment has been satisfied, released, or discharged; a prior judgment on which it is based has been reversed or otherwise vacated; or it is no longer equitable that the judgment should have prospective application; or

(vi) any other reason justifying relief from the operation of the judgment.

Timing?

A

For grounds (i), (ii), and (iii), the motion must be made within a reasonable time not to exceed one year from the judgment;

for the other grounds, the motion must be made within a reasonable time. (But remember that a lack of subject matter jurisdiction may be raised at any time.)

Such a motion is left to the trial judge’s discretion, and, on appeal, her decision will be reviewed on an “abuse of discretion” standard.

i.e.

Here, the name and address of the witness could have been easily discovered had the pedestrian’s attorney simply requested a copy of the police report from the police department. This is such an obvious case of lack of due diligence on the part of a represented party that it would be an abuse of discretion to grant relief based on newly discovered evidence.

185
Q

Absent a court order, no execution on a judgment is allowed for

A

30 days after entry except for injunctions and receiverships, which are not held up unless otherwise ordered by a court.

186
Q

The most typical way in which compulsory joinder is raised is by a Rule 12(b) motion to dismiss for failure to join an indispensable party.

A

To determine if the party is truly indispensable, the first step is to determine whether the absentee should be joined. One reason why an absent party should be joined is when complete relief cannot otherwise be accorded among parties. However, under the tort law in a number of states, any single joint tortfeasor may be liable for the entire wrong, making complete relief for the plaintiff possible. Another reason to join an absent party is if he has an interest in the subject matter of the suit such that, without his presence, his ability to protect his interest will be impaired or the existing parties will be subject to a substantial risk of incurring multiple or inconsistent obligations.

Here, the electrician has no interest to protect. Even if the condo owner were to sue the electrician later, the electrician would not be precluded from raising any defenses that the gas worker raised, because the electrician was not a party to the initial case and therefore issue preclusion would not apply to him. Accordingly, the electrician is not a necessary party.

187
Q

Federal Rule of Civil Procedure 15 states that leave of court (to grant motions to amend) is to be “freely given when justice so requires.”

A

The rule does not provide any clear date when amendments are no longer permissible, although later amendments obviously would be less fair and less likely to be considered in the interest of justice.

Additionally, for statute of limitations purposes, proposed claims may be considered to “relate back” to the date of the original pleading in which the claim was made under Rule 15(c).

That is critical here because, although the statute of limitations for the proposed products liability claim had expired at the time the motion was made, the amended complaint asserting a products liability claim relates back to the original filing because that claim stems from the same facts alleged in the original complaint, and the statute of limitations had not expired at the time of the original filing.

(A) is incorrect because a plaintiff may amend the complaint once as a matter of course (i.e., without court intervention) not later than 21 days after service of the auto manufacturer’s answer. Here, 40 days have passed since the auto manufacturer filed its answer, so amending as a matter of course is not permitted.

188
Q

In addition to sufficient minimum contacts with the forum state, personal jurisdiction must not offend “traditional notions of fair play and substantial justice.” The Supreme Court in International Shoe listed several factors relevant to assess the fairness factor, including the burden on the defendant in terms of convenience in defending the action.

A

Note that this still may be a losing argument, as this argument will not prevail unless the burden to the defendant is “so gravely difficult and inconvenient that a party is unfairly put at a severe disadvantage in comparison to his opponent.”

189
Q

A defendant must file a motion to dismiss based on lack of personal jurisdiction at the time he

A

files a motion or at the time he files his answer, whichever occurs first. If he does not, the defendant waives the defense.

190
Q

A partnership is a citizen of

A

each state of which one of its partners is a citizen, both limited and general.

Here, the plaintiff is a citizen of State D, and the partners are citizens of State A, State B, and State C. Thus, complete diversity exists, and the amount in controversy is stated to exceed $75,000. As a result, diversity of citizenship jurisdiction exists. A corporation’s principal place of business is one of its possible citizenships—every state in which the corporation was incorporated being the other possible citizenships. The same rule does not apply to partnerships; a partnership takes on the citizenships of its partners.

191
Q

The worker is from State B and the power company is a State A corporation with all its operations (and therefore its principal place of business) in State A. The plaintiff’s claim is for $500,000, satisfying the amount in controversy requirement.

The owner-operator shares state citizenship with the worker and was made a party when the power company impleaded him on a claim for indemnity.

Accordingly, there is diversity of citizenship jurisdiction over this claim. Once one claim satisfies the requirements for original federal subject matter jurisdiction, the court has discretion to exercise supplemental jurisdiction over related claims that derive from the same common nucleus of fact and are such that a plaintiff would ordinarily be expected to try them in a single judicial proceeding.

However,

A

for cases based solely on diversity, supplemental jurisdiction is not available for claims against persons made parties under the impleader rules when use of supplemental jurisdiction would be contrary to the requirements of diversity jurisdiction.

In the instant case, the owner-operator shares state citizenship with the worker and was made a party when the power company impleaded him on a claim for indemnity. Because a claim by the worker against the owner-operator would circumvent the complete diversity requirement, supplemental jurisdiction is not available for that claim. Hence, (A) is correct.

(B) is incorrect because supplemental jurisdiction cannot be used to override the requirements of diversity jurisdiction, as discussed above.

192
Q

When a claim is in federal court under federal question jurisdiction, and the plaintiff has a state law claim against the defendant that cannot invoke diversity jurisdiction, the federal court has discretion to

A

exercise supplemental (pendent) jurisdiction over the state law claim if the federal and state claims derive from a common nucleus of operative fact and are such that a plaintiff would ordinarily be expected to try them all in one judicial proceeding. The court may continue to exercise supplemental (pendent) jurisdiction over the state claim even though the federal claim is dismissed on the merits. However, the state claim should probably also be dismissed (without prejudice) if the federal claim is dismissed before trial. Here, although the federal copyright claim was invalid, it was deemed to be so during trial, before a verdict was rendered. Since the case is currently being tried, the court will likely hear the state claim for the sake of judicial economy.

193
Q

Although diversity of citizenship appears to be present in this action, it is only incidental. The removal statute provides that “any civil action of which the district courts have [federal question jurisdiction] shall be removable without regard to the citizenship or residence of the parties.” [28 U.S.C. §1441(b)]

A

When both a federal question and diversity jurisdiction exist, the federal question jurisdiction normally “trumps” diversity jurisdiction. In any event, if this case were to be solely based on diversity, the defendant could not remove because she is a citizen of the forum state.

194
Q

The federal courts and the state courts have concurrent jurisdiction over most types of actions. The few areas over which the federal courts have exclusive jurisdiction include

A

bankruptcy cases, patent and copyright cases, antitrust cases, and a few other less common types of actions.

195
Q

The court does not have federal question jurisdiction because the recently enacted federal statute arises only in anticipation of the defendant’s defense. This is insufficient to confer federal question jurisdiction.

A

(Rationale: If, for example, the defendant relied on some other defense or defaulted instead of defending on the basis of the new federal statute, no federal question would ever be involved in the case. [See Louisville & Nashville Railroad v. Mottley, 211 U.S. 149 (1908)])

196
Q

An expert is a person with knowledge, skill, experience, training, or education in a particular field. The opinion of an expert consulted in anticipation of litigation is discoverable if

A

a party intends to call the expert as a trial witness. Conversely, the opinion of an expert who is retained in anticipation of litigation but who is not expected to testify at trial (i.e., a consulting expert) is discoverable only upon a showing of exceptional circumstances under which it is impracticable to obtain facts or opinions by other means. Absent such circumstances, these opinions are not discoverable, regardless of whether the nontestifying expert was informally consulted or whether she was retained or specially employed.

197
Q

Generally, a party may obtain discovery regarding any matter that is proportional to the needs of the case and that is relevant to any party’s claim or defense, provided the matter is not subject to a privilege or to the exception for trial preparation materials, also called work product.

A

Trial preparation materials are documents or other tangible things prepared in anticipation of litigation or for trial.

Here, the manager’s report is relevant because it contains facts relating to the incident that is the basis of plaintiff’s claim. Moreover, it is not subject to the exception for trial preparation materials because it was prepared in the ordinary course of business to comply with state law, not in anticipation for litigation. Finally, the costs of producing the report likely would not be prohibitive.

198
Q

Assuming the interrogatory is otherwise proper, the manufacturer may search the 2 million order forms itself or it may allow the collector access to the files.

A

In a situation where desired information may be ascertained from the business records of the party on whom the interrogatory was served, and where the burden of finding the information is substantially the same for the party serving the interrogatory as for the party served, it is a sufficient answer to provide the serving party reasonable opportunity to examine the records. [Fed. R. Civ. P. 33]

199
Q

A transfer on the ground that the original choice of venue was improper generally results in a change of the law applicable under the Erie doctrine; i.e.,

A

i.e., the law of the state in which the transferee court sits now applies.

This is in contrast to a transfer on convenience grounds, in which case the law of the transferor court continues to apply.

200
Q

Even if the court has personal jurisdiction over the defendants, subject matter over the action, and venue is proper,

A

the court has discretion to decline to hear the case in deference to a court in another country if it determines that the other country provides a substantially more efficient, convenient, and fair forum.

201
Q

A defending party may implead a nonparty if

A

the nonparty is or may be liable to the party for any part of a judgment that the plaintiff may recover against it.

Because the exterminator is claiming that the subcontractor is liable for the exterminator’s liability to the plaintiff, it may implead the subcontractor.

202
Q

Under Rule 38, to receive a jury trial a party must

A

file a written demand and serve it on all the parties within 14 days after the service of the last pleading directed to the jury-triable issue. Otherwise, the right to a jury trial generally is deemed waived. Note, however, that the Supreme Court has held that, in the absence of compelling reasons to the contrary, a court should grant relief from the waiver if the issue is normally tried by a jury.

203
Q

Pursuant to Rule 12(b), improper venue must be raised

A

in a defendant’s first response—either in its timely motion to dismiss before the answer or in the answer, whichever is first.

204
Q

The issue is whether the court may transfer the case from one proper venue to another.

A

Such a transfer has no strict time limit. Rather, transfer is left to the discretion of the trial judge, and the judge may refuse transfer where the case has been pending for some time and would work a prejudice to one of the parties. Two months is likely not a sufficient period to prevent transfer of the case.

205
Q

When a court grants summary judgment on some but not all of the claims in an action, the court’s order is not final and thus not appealable unless

A

the court expressly determines that there is no just reason to delay entry of judgment.

Unless the trial judge makes that express determination, its order determining the merits of fewer than all of the claims is not a final judgment and is not appealable.

Summary judgment is not a final judgment unless it disposes of all of the parties’ claims or the court expressly determines that there is no just reason for delay.

206
Q

Because the aerospace company did not timely object to the instructions, the court will review them using a

A

plain error standard of review.

In order to reserve its right to appeal an error in an instruction given or, as is the case here, a failure to give an instruction, a party must object on the record before the instructions are given. If adequately preserved, the instructions are reviewed using an abuse of discretion standard. However, if the objection is not made (and therefore not preserved for full appellate review), then the court’s review is limited to considering whether there was a plain error in the instruction that affected substantial rights.

207
Q

When an appellate court reviews a trial court’s ruling on a post-trial motion for judgment as a matter of law (including a renewed one), it employs

A

a de novo standard.

It does so because the issue is one of law.

In contrast, when an appellate court reviews a trial court’s denial of a motion for new trial, it employs a more deferential standard, reversing the trial court’s denial only when there is a clear showing of an abuse of discretion.

Note that when a renewed motion for judgment as a matter of law and a motion for a new trial are made in the alternative and the renewed motion is granted, the court rules hypothetically on the new trial motion so that no remand is required if the ruling on the judgment as a matter of law is subsequently reversed on appeal.

208
Q

In a properly removed case, venue is proper in the federal court of the state where the case was pending, even if venue would have been improper had the plaintiff originally filed the action in the federal district court of that state.

A

This is because venue for an action removed under section 1441(a) lies in the federal district court “embracing the place where such [state] action is pending.”

209
Q

. There are essentially two time restrictions on removal of a diversity case to federal court:

A

(1) a case based on diversity must be removed within 30 days of the defendant’s receipt of a copy of the paper (order, motion, etc.) that makes the case removable; but

(2) in no event may the case be removed more than one year after it was commenced in state court. [28 U.S.C. §1446]

210
Q

Although both merger and bar are used to indicate that claim preclusion (res judicata) is in effect, where the claimant won the earlier lawsuit, the claim is said to

Where the defendant won the earlier lawsuit, the claim is said to

A

be merged into the prior judgment.

be barred by the prior judgment.

Because the question specifically states that the claimant won the earlier suit, to say that the claim either merged into or is barred by the prior judgment is incorrect.

211
Q

While driving in a city in State A, a citizen of State B struck a pedestrian who was a citizen of State C. The pedestrian sued both the driver and the State A city in federal district court, seeking $100,000. The pedestrian alleged that the driver was liable for negligently operating his car and that the State A city was liable for negligently maintaining a traffic signal. The driver, who owns an office supply wholesaler, also has a contract claim against the State A city for $80,000 worth of furniture that the city allegedly purchased and received but never paid for. Assume that State A has waived any applicable governmental immunity.

If the driver files a cross-claim for negligence against the State A city to recover damages for his injuries in the accident with the pedestrian, can the driver join with that cross-claim his contract claim for the purchase price of the furniture and maintain the claim in the same federal action?

A

The driver can join his contract claim with his negligence cross-claim, but he is not required to do so.

As a general rule, a party may assert a cross-claim against a co-party only if the cross-claim arises from the same transaction or occurrence as that of the original action or of a counterclaim. However, once the party has filed such a cross-claim, he also may join with it any other claim that he has against the same party.

Here, the driver’s contract claim is unrelated to the pedestrian’s negligence claim; however, the driver’s negligence claim is related to the pedestrian’s negligence claim. Thus, the contract claim also can be asserted in this case.

212
Q

A manufacturer of widgets was incorporated in and has its principal place of business in State A. The manufacturer also operates its own stores in State A, State B, and State C, through which it sells its widgets to consumers. A consumer who resides in State D visited a State D store to purchase a widget. The State D store did not have the widget he needed, and employees at the store directed him to the State B store operated by the manufacturer. The consumer visited the manufacturer’s State B store and purchased the widget. The consumer returned to his home in State D, where he was injured while using the widget. The consumer intends to file a products liability action against the manufacturer in federal district court.

In what district or districts is venue proper?

A

Venue is proper in the District of State A, the District of State D, and the District of State B.

Federal venue in civil actions is proper in

(i) the district where any defendant resides, if all defendants are residents of the state in which the district is located; and

(ii) the district in which a substantial part of the events or omissions giving rise to the claim occurred.

The District of State D is proper because a substantial part of the events giving rise to the claim—use of the widget and the injury—took place there. Substantial events also occurred in the District of State B (the sale). Additionally, venue is proper in other districts because the manufacturer is deemed to reside there.

Corporations are deemed to reside in any district in which it is subject to personal jurisdiction with respect to the action in question.!!!!!!

The manufacturer is subject to personal jurisdiction and thus deemed to reside in the District of State A (because its incorporation and principal place of business are there, thus subjecting it to general jurisdiction there), the District of State B (because it sold the widget from its store there, thus subjecting it to specific jurisdiction there with respect to the current action), and the District of State D (because the injury took place in State D, thus subjecting it to specific jurisdiction there with respect to the current action).

213
Q

Using his cellphone, a witness recorded a speeding driver hitting a pedestrian. The witness sold the recording to the driver. The driver then gave the recording to his attorney. After the pedestrian filed suit against the driver, the pedestrian sent a discovery request to the driver requesting that he produce “all items that show or describe the accident.”

Which of the following best describes whether the driver must provide the video, or a copy of it?

A

The driver must provide the video.

Parties are entitled to discovery that fits under Rule 26(b) (1), which includes “any nonprivileged matter that is relevant to any party’s claim or defense.” In other words, even if the matter is only relevant to the opposing party, it would still be covered. Additionally, Rule 34 requires a party to produce relevant physical material, including electronically stored information, such as the recording here. There is no exception to relevance for matter that is damaging to a party, so (B) is incorrect. (Being damaging to the defendant’s defense might be reason not to disclose it as an initial disclosure because the defendant would not use the recording to support his defense, but the recording would have to be disclosed on a proper request.)

214
Q
A